Anda di halaman 1dari 117

Sample Calculus Problems

Part 1: Single Variable Functions

Part 2: Multi-Variable Functions

58

Part 3: Sequences and Series

90

Part 4: Vector Analysis

109

Last revision: August 27, 2014

[This

page is intentionally left blank.

Part 1: Single Variable Functions

1.

Evaluate the limit

Solution:

Remark:

4 3x + 1
lim 2
x5 x 7x + 10

. (Do not use L'Hpital's Rule.)

4 3x + 1
(4 3x + 1)(4 + 3x + 1)

lim 2
= lim
x5 x 7x + 10
x5 (x2 7x + 10)(4 +
3x + 1)
16 (3x + 1)

= lim
x5 (x2 7x + 10)(4 +
3x + 1)
15 3x

= lim
x5 (x 5)(x 2)(4 +
3x + 1)
3(5 x)

= lim
x5 (x 5)(x 2)(4 +
3x + 1)
3

= lim
x5 (x 2)(4 +
3x + 1)
3

=
(5 2)(4 + 3 5 + 1)
3
=
38
1
=
8

 =" is the most frequently used verb in mathematics. It was introduced in 1557 by

Robert Recorde to avoid the tedious repetition of the words `is equal to'." It is important to
use the equal sign correctly.

To introduce  ", the phantom equal sign, to avoid the tedious repetition of the symbol
 =" is not a good idea. The solution above should not go like:

4 3x + 1
lim 2
x5 x 7x + 10

(4 3x + 1)(4 + 3x + 1)

lim
x5 (x2 7x + 10)(4 +
3x + 1)
15 3x

lim
x5 (x 5)(x 2)(4 +
3x + 1)
3(5 x)

lim
x5 (x 5)(x 2)(4 +
3x + 1)

One must also not use other symbols, which have completely dierent meanings, in place
of  =". The solution above should not go like:

4 3x + 1
(4 3x + 1)(4 + 3x + 1)

lim 2
lim
x5 x 7x + 10
x5 (x2 7x + 10)(4 +
3x + 1)
15 3x

lim
x5 (x 5)(x 2)(4 +
3x + 1)
3(5 x)

lim
x5 (x 5)(x 2)(4 +
3x + 1)

The equal sign always stands between two things, although sometimes one of these things
are at the end of the previous line or at the beginning of the next line. The solution above
should not start like:

This begs the question: What is

(4 3x + 1)(4 + 3x + 1)

= lim
x5 (x2 7x + 10)(4 +
3x + 1)

(4 3x + 1)(4 + 3x + 1)

equal to lim
?
x5 (x2 7x + 10)(4 +
3x + 1)

The equal sign can be used between two functions when we deal with identities , like

x2 1
=x+1
x1

for all

x =/ 1

or when we deal with equations , like

Find all

such that

x2 = 4.

Therefore we can not just drop some of the limit signs in the solution above to make it
look like:

4 3x + 1 (4 3x + 1)(4 + 3x + 1)

lim 2
=
x5 x 7x + 10
(x2 7x + 10)(4 + 3x + 1)

=
(x 2)(4 + 3x + 1)
3

=
(5 2)(4 + 3 5 + 1)
3
=
38
1
=
8

The equalities on the lines marked with


not equal to

1
8

are not correct.

because, for instance, if we let

3
1
1

= =/ .
8
(1 2)(4 + 3 1 + 1) 2
2

x = 1

then

is
(x 2)(4 + 3x + 1)
3

=
(x 2)(4 + 3x + 1)

2.

Let

Express

be the slope of the tangent line to the graph of


as a limit. (Do not compute

Solution:

(x0 , f (x0 ))

The slope

x2
x+2

y =

m.)

at the point

y = f (x)

of the tangent line to the graph of

(3, 9).

at the point

is given by the limit

f (x) f (x0 )
xx0
x x0

m = lim
or equivalently by the limit

f (x0 + h) f (x0 )
.
h0
h

m = lim
Therefore two possible answers are

x2
(3 + h)2
(9)
(9)
x
+
2
3
+
h
+
2
m = lim
= lim
.
x3
h0
x (3)
h

3.

Suppose that

Solution:

lim f (x) =/ 0

and

lim g(x) = 0.

Assume that

lim

f (x)
g(x)

xc

xc

xc

Show that

exists, and let

f (x)
xc g(x)

lim

L = lim
xc

f (x)
.
g(x)

does not exist.

Then by the product

rule for limits we obtain

lim f (x) = lim (


xc

xc

f (x)
f (x)
g(x)) = lim
lim g(x) = L 0 = 0 .
xc
g(x)
g(x) xc

This contradicts the fact that


true:

4.

lim
xc

f (x)
g(x)

lim f (x) =/ 0.
xc

Therefore our assumption cannot be

does not exist.

lim f (x) = 0 and there exists a constant K such that g(x) K


xc
interval containing c. Show that lim (f (x)g(x)) = 0.

Suppose that

some open

Solution:

for all

x =/ c

xc

We have

f (x)g(x) = f (x) g(x) f (x) K

in some open interval

around c. Therefore

Kf (x) f (x)g(x) Kf (x) .


Now applying the Sandwich Theorem and using the fact that
the result.

lim f (x) = 0 we obtain


xc

in

5.

Determine the following limits if

a.

x0

d.

x0

lim f (x) = A

x0+

lim f (x2 x)

b.

x0

lim (f (x3 ) f (x))

e.

x1

and

lim f (x) = B .

x0

c.

lim (f (x2 ) f (x))

lim f (x3 x)

x0+

lim f (x2 x)

Solution: a.

If x < 0, then x2 > 0 and x > 0. Therefore x2 x > 0 for x < 0, and
2
x x approaches 0 from the right as x approaches 0 from the left. lim f (x2 x) = A.
x0

b.

Since

x2 > 0

the left. Hence

c.

For

as

d.

Since

left.

e.
x

6.

Let

(x 1)2

x < 0, x2 approaches 0 from the right as x approaches


lim (f (x2 ) f (x)) = lim f (x2 ) lim f (x) = A B .
for

x0

0 < x < 1,

x0

we have

x3 < x

x0

x3 x < 0. So x3 x approaches
3
Therefore lim f (x x) = B .
+

and

approaches 0 from the right.

0 from the left

x0

x3 < 0 for x < 0, x3 approaches 0 from the left as x approaches


3
3
Hence lim (f (x ) f (x)) = lim f (x ) lim f (x) = B B = 0.

For

x0

0<x<1

x0

we have

x2 < x

approaches 1 from the left.

Q be the point
+ y 2 = 1 and the

0 from

0 from the

x0

x2 x < 0. x2 x approaches
2
Hence lim f (x x) = B .

and

0 from the left as

x1

of intersection in the rst quadrant of the circle C1 with equation


circle C2 with equation x2 + y 2 = r 2 . Let R be the point where the line

passing through the points


as r 0+ .

P (0, r)

and

intersects the

x-axis.

Determine what happens to

Solution:

x2 + y 2 = r2 from (x 1)2 + y 2= 1 we obtain x = r2 /2,


in x2 + y 2 = r 2 gives us Q(r 2 /2,
r2 r4 /4).

Subtracting

substituting this back

and

R(a, 0) be the coordinates of R and let S be the foot of the perpendicular from
Q to the x-axis. Since the triangles RSQ and ROP are similar we have
Let

a r2 /2
a

=
r2 r4 /4 r
and hence

a=

r3 /2

.
r r2 r4 /4

Then

lim+ a = lim+

r0

r0

r3 /2

r r2 r4 /4

r3 /2

(r
+
r2 r4 /4))
r0
r2 (r2 r4 /4)

= 2 lim+ (1 + 1 r2 /4)
r0

= 2 (1 + 1 02 /4) = 4 .
= lim+ (

Therefore

7.*

approaches the point

(4, 0)

as

r 0+ .

Use the formal denition of the limit to show that

Solution:

Given

>0

we want to nd

>0

lim

x1/2

1
= 2.
x

such that

1
2 < .
x

1
0 < x <
2

()

We will do this in two dierent ways.

Solve the Inequality" Method : First we solve

1
2 <
x

for

x.

1
1
2 < 2 < < 2 +
x
x
The next step depends on whether
If

2 > 0,

that is if

< 2,

is positive, zero or negative.

then

2<

1
1
1
< 2 +
>x>
.
x
2
2+

*Examples marked red are not part of the Fall 2014 syllabus.

If

2 = 0,

that is if

= 2,

then

2<

If

2 < 0,

that is if

> 2,

2<

1
1
< 2 + x >
.
x
2+

then

1
1
< 2 + x >
x
2+

or

1
>x.
2

Next we choose

in such a way that every

lies in the solution set of

1
2 < ,
x

and therefore the implication

all three cases choosing a delta such that

1
0 < x <
2
in () holds. In

satisfying the condition

0 < 1/2 1/(2 + ) = /(4 + 2)

achieves

this.

The Estimation Method : Suppose

0 < x 1/2 <

for some

> 0.

Then

1
2x 1/2 2
2 =
<
.
x
x
x
to satisfy 1/4. (Why 1/4?) Then
1/2 < x < 1/2 + 1/4 < x < 3/4 4 > 1/x > 4/3

At this point let us also decide to choose

0 < x 1/2 <


1/x < 4 and therefore

1
2
2 <
< 8 .
x
x

> 0 if we choose to satisfy /8 (as well as 1/4) then


1

we will have 2 < 8 8


= and () will hold. In conclusion, any choice of
x
8
satisfying 0 < min{/8, 1/4} works.

Hence for a given

8.* Show that x3


lim (x4 + 7x 17) = 43 using the formal denition of the limit.
Solution:

For any given

>0

we have to nd a

0 < x (3) <

>0

so that for all

x4 + 7x 17 43 < .

*Examples marked red are not part of the Fall 2014 syllabus.
7

we have

(x4 + 7x 17) 43 = x4 + 7x 60 = (x + 3)(x3 3x2 + 9x 20). Suppose


that 0 < x (3) < and 1. Then 4 3 < x < 3 + 2. In particular,
x < 4. Therefore, using the Triangle Inequality, we obtain x3 3x2 + 9x 20
x3 + 3x2 + 9x + 20 < 43 + 3 42 + 9 4 + 20 = 168. Now if we choose to satisfy
0 < min{/168, 1}, then we have

x4 + 7x 17 43 = x4 + 7x 60 = x + 3 x3 3x2 + 9x 20 < 168


168 =
168
whenever 0 < x (3) < . We are done.
We have

9.* Suppose that for all 0 < < 1,


x 1 <

2
4

f (x) 3 <

x 1 <

35

g(x) 4 < .

and

Find a real number

>0

If we take

1
10

in

1
10

1
.
5

1
400

f (x) 3 <

1
.
10

1
350

g(x) 4 <

1
.
10

() we get

in

x 1 <
Therefore if

f (x) + g(x) 7 <

() we get

x 1 <
If we take

()

such that

x 1 <

Solution:

()

1
,
400

x 1 <

then we have

f (x) + g(x) 7 = f (x) 3 + g(x) 4 f (x) 3 + g(x) 4 <


by the Triangle Inequality. Hence we can take

Remark:

10.*

Let

In fact, any

f (x) =

a.

Show that if

b.

Show that

94 2

1225

if

x=

1
n

works.

where

is a positive integer,

otherwise.

c =/ 0

then

lim f (x)
x0

1
.
400

lim f (x) = 0.
xc

does not exist.

*Examples marked red are not part of the Fall 2014 syllabus.
8

1
1 1
+
=
10 10 5

Solution: a.
the closest to

Assume

c > 0.

Then there is a positive integer

among all real numbers dierent from

is a positive integer. (Why?) Let

0 < x c <

x =/

1
n

= c

1
> 0.
m

Therefore

Assume

Take

= c.

Then for any

> 0,

x =/

lim f (x) = 0.

b.

be a real number and assume that

>0

such that for all

f (x) 0 = 0 0 = 0 < .

positive integer

f (x) 0 = 0 0 = 0 < .

is not 0, let

= L/2 > 0.

x = /2.

Then

Then for every

> 0,

0 < x <

L = 0 L = f (x) L < L/2

is not

x0

f (x) L < .

Then there is a

0 < x <

lim f (x) = L.

x,

0 < x 0 <

Take

we have

xc

there exists a

is

where

we have

1
for any
n
f (x) = 0

x<0

Therefore

If

> 0,

1/n

xc

c < 0.

1/m

such that

lim f (x) = 0.

0 < x c <

Let

and of the form

Then for any

for any positive integer

f (x) = 0

>0

such that

f (x) L < L/2 .

is true, but

true. We have a contradiction.


On the other hand, if

L = 0,

let

= 1/2.

0 < x <
If

but

is a positive integer satisfying

1 = 1 = f (x) < 1/2

Hence

11.

lim f (x)
x0

Then there is a

>0

such that

f (x) < 1/2 .

n > 1/ ,

take

x = 1/n.

Then

0 < x <

is true,

is not true. Again we have a contradiction.

cannot exist.

Show that the equation

x2 10 = x sin x

has a real solution.

Solution:

f (10) =

Consider the function f (x) = x2 10 x sin x. Then f (0) = 10 < 0 and


2
10 10 10 sin(10) = 90 10 sin(10) 90 10 = 80 > 0. Note that f is

continuous on
the function

in

( 0, 10)

[ 0, 10].

Therefore we can apply the Intermediate Value Theorem to

[ 0, 10] for the value 0 to conclude that there is a point


f (c) = 0. This c is also a solution of the given equation.

on the interval

such that

12.

Consider the equation

x2
= cos x .
4

a.

Show that this equation has at least one real solution.

b.

Show that this equation has at least two real solutions.

c.

Show that this equation has at least three real solutions.

Solution:

Let

f (x) = 1

correspond to the zeros of


As

f (0) = 0, x = 0

Now observe that

x2
cos x.
4
f.

The solutions of the equation

x2
= cos x
4

is one zero.

f (/2) = 1 2 /16 > 0

f () = 2 2 /4 < 0

and

as

4 > > 3.

As

is continuous on the entire real line, applying the Intermediate Value Theorem to
the function

[/2, ]

on the interval

interval such that

f (c) = 0.

Finally, as the function

we conclude that there is a point

in this

This is our second zero.

is even, we have

f (c) = f (c) = 0,

and

x = c

is our third

zero.

13.

Show that at any moment there are two antipodal points on the equator of the Earth with

the same temperature.

Solution:

First we will make a mathematical model of the problem.

We will

. We
in radians,
= 7/4, and

consider the equator as a circle, and use the longitude as our coordinate
choose the positive direction for
and let it take any real value.

= 15/4,

among other values.

the point corresponding to

T ( + 2) = T ()

for all

want to show that there is a

to correspond to the East, measure


So 45 W corresponds to = /4,
Note that

We let

T ()

T is a continuous
T (c + ) = T (c).

We will assume that

such that

corresponds to the antipode of

denote the temperature at

We have

function. We

f () = T ( + ) T (). Note that since T is continuous, f is


continuous. Our quest to nd a c such that T (c + ) = T (c) is equivalent to nd a
c such that f (c) = 0. If f (0) = 0, then we let c = 0 and we are done. Suppose that
f (0) =/ 0. Observe that f (0) = T () T (0) = T () T (2) = f (). In other words,
f (0) and f () have opposite signs. Now we apply the Intermediate Value Theorem
to f on the interval [0, ] for the value 0, and conclude that there is a point c in
[0, ] such that f (c) = 0. We are done.
Consider the function

Remark:

It is possible to show that at any moment there are two antipodal points on Earth

with the same temperature and the same pressure.

10

14.

Find all tangent lines to the graph of

y = x3

that pass through the point

(2, 4).

Solution:

As dy/dx = d(x3 )/dx = 3x2 , the equation of the tangent line through
3
3
2
a point (x0 , x0 ) on the graph is y x0 = 3x0 (x x0 ). This line passes through
(2, 4) exactly when 4 x30 = 3x20 (2 x0 ), or in other words, x30 3x20 + 2 = 0. We

x0 = 1 is a root of this polynomial. Therefore we have the factorization

(x0 1)(x20 2x0 2). The roots of the quadratic factor are x0 = 1 3.

Therefore the tangent lines to y = x3 at the points (1, 1), (1 +


3,
10
+
6
3), and

(1 3, 10 6 3) pass through
(2,
4)
. The equations of these lines are y = 3x 2,

y = (12 + 6 3)x (20 + 12 3), and y = (12 6 3)x (20 12 3), respectively.
observe that
x30 3x20 + 2 =

15.

Evaluate the limit

Solution:

lim
x0

lim
x0

1 + sin2 x2 cos3 x2
.
x3 tan x

1 + sin2 x2 1 1 cos3 x2 x
1 + sin2 x2 cos3 x2
+

= lim
x0
x3 tan x
x4
x4
tan x
sin x2 2
1
x
1 cos3 x2
= lim ((( 2 )
)
+
)
4
2
x0
x
x
tan x
1 + sin x2 + 1

Now we observe that:

sin x2
=1
x0 x2
1
1
1
lim
=
=
2 2
2 2
x0
1 + sin x + 1
1 + sin 0 + 1 2
x
lim
=1
x0 tan x
lim

11

and

1 cos3 x2
1 cos x2
=
lim
(
(1 + cos x2 + cos2 x2 ))
x0
x0
x4
x4
2 sin2 (x2 /2)
(1 + cos x2 + cos2 x2 ))
= lim (
x0
x4

lim

1
sin(x2 /2)
= (lim
) lim(1 + cos x2 + cos2 x2 )
x0
2 x0 x2 /2
1
= 12 3
2
3
= .
2

1 + sin2 x2 cos3 x2
1 3
lim
= (1 + ) 1 = 2 .
3
x0
x tan x
2 2

Therefore:

16.

Find the equation of the tangent line to the graph of

x = 1.

y = sin2 (x3 /6)

at the point with

dy
= 2 sin(x3 /6) cos(x3 /6) 3x2 /6 .
dx
3
dy
= 2 sin(/6) cos(/6) /2 =
.
dx x=1
4

Solution:

Since

y = sin2 (x3 /6)

yx=1 = 1/4,

using the point-slope formula we nd the equation of the tangent

line as

1
y =
4

3
(x 1)
4

or, after some reorganization,

y=

17.

Let

3
1 3
x+
.
4
4

2x + x2 sin( )

f (x) =

a.

Find

b.

Show that

f (x)

for all

Therefore,

x.

is not continuous at

0.

12

if

x =/ 0,

if

x = 0.

Solution: a.

For

f (x) =
for

x =/ 0.

For

x=0

x =/ 0 we compute the derivative using the rules of dierentiation:

d
(2x + x2 sin(1/x)) = 2 + 2x sin(1/x) + x2 cos(1/x) (1/x2 )
dx

we must use the denition of the derivative:

f (0 + h) f (0)
2h + h2 sin(1/h)
= lim
h0
h0
h
h
1
= lim 2 + lim h sin( ) = 2 + 0 = 2
h0
h0
h

f (0) = lim

lim h sin(1/h) = 0

whose proof uses the Sandwich (or


h0
Squeeze) Theorem. Here is a recap of the proof: Since sin(1/h) 1 for all h =
/0
Here we used the fact that

we have

h sin(1/h) = h sin(1/h) h

for all

1
h h sin( ) h
h
As

lim h = 0 = lim(h),
h0

lim h sin(1/h) =
h0

h0
0.

To summarize:

b.

it

follows

h =/ 0.

for all

by

the

1
1

2 + 2x sin( ) cos( )

x
x

f (x) =

lim f (x).

h =/ 0 .
Sandwich

if

x =/ 0,

if

x = 0.

Theorem

that

(a)

and lim 2x sin(1/x) = 0 as in part


.
x0
x0

However lim cos(1/x) does not exist. It follows that lim f (x) does not exist and
x0
x0
hence f is not continuous at 0.
Consider

x0

We have

lim 2 = 2

Therefore

13

18.

Find

d2 y

if y is a dierentiable function of x satisfying the equation x3 +2y 3 = 5xy .


dx2 (x,y)=(2,1)

Solution:
x3 + 2y 3 = 5xy

d/dx

3x2 + 6y 2

dy
dy
= 5y + 5x
dx
dx

()

x = 2, y = 1

12 + 6

dy
dy
= 5 + 10
dx
dx

dy
=7
dx

dy 7
=
dx 4

at

(x, y) = (2, 1)

Now we dierentiate the equation marked

14

()

with respect to

to nd the second

derivative.

3x2 + 6y 2

dy
dy
= 5y + 5x
dx
dx

d/dx

dy
d2 y
dy
dy
d2 y
6x + 12y ( ) + 6y 2 2 = 5
+5
+ 5x 2
dx
dx
dx
dx
dx

x = 2, y = 1,

dy 7
=
dx 4

d2 y
7
d2 y
7
12 + 12 ( ) + 6 2 = 10 + 10 2
4
dx
4
dx

d2 y 125
=
dx2
16

Remark:

An alternative approach is to solve

dierentiate this with respect to

19.

(x, y) = (2, 1)

at

from

(),

viz.

y =

y .

to nd

5y 3x2
,
6y 2 5x

and then

A piston is connected by a rod of length 14 cm to a crankshaft at a point 5 cm away from

the axis of rotation of the crankshaft. Determine how fast the crankshaft is rotating when the
piston is 11 cm away from the axis of rotation and is moving toward it at a speed of 1200
cm/sec.

Solution:

Let

P (x, y)

and

Q(a, 0)

be the ends of the connecting rod as shown in

the picture. The axis of rotation of the crankshaft passes through the origin of the

xy -plane

and is perpendicular to it.

The point

where the rod is connected to

crankshaft moves on a circle with radius 5 cm and center at the origin. The point

Q where the rod is connected to the piston moves along the positive x-axis.
angle between the ray OP and the positive x-axis.
y
P (x, y)

14 c
m

Q(a, 0)
x

5
x2 + y 2 = 25

The question is:

a = 11 cm

and

da
= 1200 cm/sec
dt
15

d
=?
dt

is the

We have

x2 + y 2 = 52

(1)

and

(x a)2 + y 2 = 142 .

(11)

At the moment in question a = 11 cm. Substituting this in (1) and (11) we obtain
x2 + y 2 = 52 and (x 11)2 + y 2 = 142 . Subtracting the second equation from the
rst
2
2
2
gives 22x11 = 5 14 , and solving for x we get x = 25/11 cm. Then y = 20 6/11
cm.
Dierentiating (1) and (11) with respect to time

x
and

(x a) (
At the moment in question

y = 20 6/11

a = 11

we obtain

dx
dy
+y
=0
dt
dt

(111)

dx da
dy
)+y
=0.
dt dt
dt

(18)

cm,

da
= v = 1200
dt

cm/sec,

x = 25/11

cm and

cm. Substituting these in (111) and (18) we get

dy
dx
+4 6
=0
dt
dt

(8 )

and

dy
dx
+ 20 6
= 146v .
(81)
dt
dt
dx
dx 146
Subtracting 5 times (8) from (81) we nd 121
= 146 v , and hence
=
v.
dt
dt 121
dy
365
v.
Substituting this back in (8) gives
=
dt 242 6
146

Now we are ready to compute

d
.
dt

Since

tan = y/x,

dierentiation gives

dy
dx
x y
d
sec2
= dt 2 dt
dt
x
and using

sec2 = 1 + tan2 = 1 + (y/x)2

we obtain

dy
dx
d x dt y dt
.
=
dt
x2 + y 2

25
20 6
dx 146
Plugging x =
cm, y =
cm,
=
v,
11
11
dt 121

d
73
1460 6
v=
gives
=
radian/sec.
dt
11
110 6

16

and

dy
365
v
=
dt 242 6

in this formula

Remark:
Remark:

2 5 x cos

1460 6
11

radian/sec is

1460 6 60

11
2

rpm or approximately 3105 rpm.

This problem has a shorter solution if we use the law of cosines. Start with
= 142 and dierentiate with respect to t to obtain

x2 + 52

d 5 cos 11
=
v.
dt
5 sin
Put

x = 11

cm in the rst equation to nd

cos = 5/11

and then

sin = 4 6/11.

Now

substituting these in the second equation gives the answer.

20.

Determine how fast the length of an edge of a cube is changing at the moment when the
length of the edge is 5 cm and the volume of the cube is decreasing at a rate of 100 cm3/sec.

Solution:

a denote the length of an edge of the cube, and V denote the volume
of the cube. Then we have V = a3 . Dierentiating with respect to time t gives
dV
da
dV
= 3a2
. Substituting
= 100 cm3/sec and a = 5 cm for the moment in
dt
dt
dt
da
4
question, we obtain
= cm/sec. Therefore the length of the edge is decreasing
dt
3
4
at a rate of
cm/sec at that moment.
3

21.

Let

We measure the radius and the height of a cone with 1% and 2% errors, respectively. We

use these data to compute the volume of the cone. Estimate the percentage error in volume.

Solution:

r , h,

Let

and

be the radius, the height and the volume of the cone,

respectively.

V =

2
r h
3

Since the error in

dV =

rhdr + r2 dh
3
3

is 1% we have

dr
1% .
r

dV
dr dh
=2
+
.
V
r
h

Similarly

dh
2% .
h

triangle inequality we obtain

dV
dr dh
dr
dh
= 2
+ 2 + 2 1% + 2% = 4% .
V
r
h
r
h

The error in volume is 4% .

17

Now using the

22.

A cone of radius 2 cm and height 5 cm is lowered point rst into a tall cylinder of radius

7 cm that is partially lled with water. Determine how fast the depth of the water is changing
at the moment when the cone is completely submerged if the cone is moving with a speed of 3
cm/s at that moment.

Solution:

Let

and

be the radius and height of the cone, respectively. Let

the depth of the water in the cylinder and let

be

be the vertical distance from the

tip of the cone to the bottom of the cylinder. Let

V0

Then

be the volume of the water.

2
4
V0 = 7 L r2 h = 72 L ( h) h = 49L h3
3
3 5
75
2

where we used the fact that

r/h = 2/5.

Now dierentiating this with respect to time

0=

gives

d
dL 4 2 dh
V0 = 49
h
.
dt
dt 25 dt

In particular at the moment when the cone is completely submerged we have


cm and

49

h=5

dL
dh
=4
.
dt
dt

On the other hand, at the same moment

h=Ly
because

dy/dt = 3

dh dL dy
=

dt dt dt

dh dL
=
+3
dt dt

cm/s as the cone is being lowered at a speed of 3 cm/s.

From these two equations we obtain

dL/dt = 4/15

cm/s. In other words, the depth

of the water is increasing at a rate of 4/15 cm/s at that moment.

18

23.

Find the absolute maximum value and the absolute minimum value of

on the interval

[1, 6].

Remark: How to nd the absolute maximum and the absolute


continuous function f on a closed interval [ a, b] of nite length:
i. Compute

f (x) = x4/3 x x1/3

minimum values of a

f .

ii. Find the critical points of


iii. Add the endpoints

and

iv. Compute the value of

f
b

in

(a, b).

to this list.

at each point in the list.

v. The largest value is the the absolute maximum and the smallest value is the absolute
minimum of

Solution:
x=0

on

[a, b].

4
1
f (x) = x1/3 1 x2/3 .
3
3

The derivative is not dened at

is a critical point. Next we solve f (x)

x = 0, therefore

= 0.

4 1/3
1
x 1 x2/3 = 0 we let z = x1/3 to obtain the equation 4z 3 3z 2
3
3
1 = 0. Since z = 1 is a root, we have the factorization 4z 3 3z 2 1 = (z 1)(4z 2 +z +1).
As the quadratic factor has no real roots, z = 1 is the only solution. Therefore
x = z 3 = 13 = 1, which belongs to the interval [1, 6], is the only other critical point.

In the equation

We have f (0) = 0, f (1) = 1, f (1) = 3, and f (6) = 5


5 61/3 6 > 3 5 61/3 > 9 53 6 > 93 725 > 721.

61/3 6.

We conclude that the absolute maximum and minimum values of


on the interval [1, 6] are 5 61/3 6 and 1, respectively.

24.

f (x) = x4/3 xx1/3

Find the absolute maximum and the absolute minimum values of

interval

f (x) =

[ 0, ).

Remark:

Observe that

x2

x+1
+x+9

on the

When looking for the absolute maximum and the absolute minimum values of a

continuous function on an interval that is not necessarily closed or of nite length, a modied
version of the algorithm above can be used.

In Step iv , if an endpoint does not belong to the interval, then we compute the appropriate
one-sided limit of the function at that point instead of the value of the function.
In Step v , if the largest value (which can be

occurs only as a limit, then we conclude

that there is no absolute maximum. Similarly for the smallest value.

Solution:
and

x = 2.

f (0) =

We compute
Only

x=2

1
1
, f (2) =
,
9
5

is in the

and

x2 + 2x 8
. The roots of f (x) = 0 are x = 4
(x2 + x + 9)2
interval [ 0, ). So our list is 0, 2, and .

f (x) =

lim f (x) = 0.

x
and there is no absolute minimum.

Since

19

1 1
> > 0,
5 9

the absolute maximum is

1
5

25.

Suppose that a function

f (x0 ) = 0

for some

Solution:

x0 > 0.

x0 > 0,

Since

f (x) = f (x/2) for all x and f (0) = 1.


x1 such that 0 < x1 < x0 and f (x1 ) = 0.

satises

then there is

is dierentiable,

is continuous. Suppose that

Show that if

f (x0 ) = 0 for some

f on [ 0, x0 ] to conclude that there


f
(x
)

f
(0)
1
0
is c such that 0 < c < x0 and f (c) =
= < 0. Then f (x) = f (x/2)
x0 0
x0
gives f (c/2) = f (c) < 0. We also have f (0) = 1 > 0. We apply the Intermediate
Value Theorem to f on [ 0, c/2] to conclude that there is x1 such that 0 < x1 < c/2
and f (x1 ) = 0. As 0 < x1 < c/2 < c < x0 we are done.

Remark:

We apply the Mean Value Theorem to

With a little bit more work it can be shown that

f (x) > 1

26.

Show that if f is a twice-dierentiable function such that f (0)


f (1) = 5, and f (x) 0 for all x, then f (x) 1/3 for all 0 x 1.

for all

x > 0.

= 1, f (0) = 1, f (1) = 2,

Solution:

Let x1 < x2 be in [0, 1]. The Mean Value Theorem applied to the

function f on the interval [x1 , x2 ] says that there is a point c in (x1 , x2 ) such that
f (x2 ) f (x1 )
= f (c). As f (c) 0 we have f (x1 ) f (x2 ). In particular, we
x2 x1
have 1 = f (0) f (x) f (1) = 5 for all x in (0, 1).

0 < x < 1.

Let

interval

[0, x], there exists a point c1

f (c1 ) 1,

f on the
f (x) f (0)
= f (c1 ). Since
x0

By the Mean Value Theorem applied to the function


in

(0, x) such that

it follows that

f (x) x + 1

( )

0 < x < 1. Similarly, applying the Mean Value Theorem to the function
f on the interval [x, 1], we see that there exists a point c2 in (x, 1) such that
f (1) f (x)
= f (c2 ). Now using the fact that f (c2 ) 5 we conclude that
1x
for

f (x) 5x 3
for

0 < x < 1.

5
[0, 1].

Adding
in

()

Remark:

times the inequality

() to the inequality () we get f (x) 1/3 for all x

In fact, it can be shown that

f (x) > 1/3

20

for all

in

[0, 1].

27.

Sketch the graph of

10 1/3 10 2/3 10 1/3


x
x =
x (1 x).
3
3
3
(, 0) and (1, ).

Solution:
y

<0

on

y = 5x2/3 2x5/3 .

y =

20
10 4/3 20 1/3
x
x
= x4/3 (x + 1/2).
9
9
9
y < 0 on (1/2, 0) and (0, ).

y =

Therefore

Therefore

y > 0

y > 0

on

(0, 1),

and

(, 1/2),

and

on

These give us the following table of signs and shapes.

x
y
y

1/2

inf.pt.

We compute the
minimum,

y -coordinates

loc.min.

loc.max.

of the important points to get

(0, 0)

for the local

(1, 3)for the local maximum,


and (1/2, 3 2) for the inection point.

3
2 > 1, we have 3 3 2 > 3. Also note that the function is continuous

Note that since


at

x = 0,

but

lim y = lim+ (

x0+

x0

and

lim y = lim (

x0
Therefore

(0, 0)

or

x = 5/2.

10 1/3
x (1 x)) = .
3

is a cusp.

Finally we nd the

x0

10 1/3
x (1 x)) =
3

x-intercepts: y = 0 5x2/3 2x5/3 = 0 2x2/3 (5 2x) = 0 x =

(The

graph is on the next page .)

21

Now we use these data to draw the graph:

Remark:

Here is the same graph drawn by Maple :

22

28.

Two corridors meet at a corner. One of the corridors is 2 m wide and the other one is 3

m wide. What is the length of the longest ladder that can be carried horizontally around this
corner?

Solution:
of

Length of the longest ladder will be equal to the absolute minimum value

in the picture.

y
2m

L
3m

x
Using the relation

3/x = y/2

we obtain

L = (x2 + 9)1/2 + (4 + y 2 )1/2 = (x2 + 9)1/2 + (4 + (6/x)2 )1/2 .


Therefore we have to minimize

2
L = (x2 + 9)1/2 (1 + )
x

for

0<x< .

We rst look at the critical points:

dL
2
2
= (x2 + 9)1/2 x (1 + ) + (x2 + 9)1/2 ( 2 ) = 0
dx
x
x
18
x + 2 (2 + 2 ) = 0 x = 181/3 m L = (22/3 + 32/3 )3/2
x
Since

lim L = and lim L = , the value at the critical point is indeed the absolute

x0+
minimum.

Hence the length of the longest ladder that can be carried around this corner is
(22/3 + 32/3 )3/2 m.

Remark:

(22/3 + 32/3 )3/2

m is approximately

7.02

23

m.

29.

Find the maximum possible total surface area of a cylinder inscribed in a hemisphere of

radius 1.

Solution:
h=

Let S be the total surface area of the cylinder,


2
1/2
(1 r ) . Hence we want to maximize

S = 2r2 + 2r(1 r2 )1/2

for

S = 2r2 + 2rh.

We have

0 r 1.

1 dS
= 2r + (1 r2 )1/2 r2 (1 r2 )1/2 = 0
2 dr
2r(1 r2
)1/2 = 2r2 1 4r2 (1 r2 ) = (2r2 1)2 8r4 8r2 + 1 = 0
8 32
1
r2 =
. Note that 2r(1 r 2 )1/2 = 2r 2 1 implies r 2
, and therefore
16
2

2+ 2
2+ 2
r2 =
and r =
.
4
2

2+ 2
We get S = (1 + 2) at the critical point r =
, S = 0 at the endpoint r = 0,
2

2 > 1 we have 1 + 2 > 2, and the absolute


and S = 2 at the endpoint r = 1. As
First we nd the critical points:

maximum value occurs at the critical point.


The maximum possible surface area of the cylinder is

24

(1 +

2) .

30.

A fold is formed on a 20 cm

30 cm rectangular sheet of paper running from the short

side to the long side by placing a corner over the long side. Find the minimum possible length
of the fold.

Solution:

ABCD be the sheet of paper and let P be the point on the edge
C is folded over. The fold runs from Q on the edge BC to R
on the edge CD . Let S be the projection of R on to the edge AB . Let L be the
length of the fold QR and let x = CQ.
AB

Let

where the corner

Q
x
A

P Q/P
B = RP /RS by the similarity
of the triangles P BQ and RSP . Hence

2
2
RP = 20x/ x (20 x) = 20x/ 40x 400 and L2 = RQ2 = RP 2 + P Q2 =
400x2 /(40x 400) + x2 = x3 /(x 10). The largest possible value of x is 20
cm. The smallest possible value of x occurs when RP = 30 cm; that is when

20x/ 40x 400 = 30 or x = 45 15 5 cm.


Then

Therefore we want to minimize


Dierentiating

where

L2 = x3 /(x 10)

2LdL/dx = 3x2 /(x 10) x3 /(x 10)2

x = 15 cm
L = 15 3 cm.
obtain

for

45 15 5 x 20.
dL/dx = 0 we
x = 15 cm we have

and setting

as the only critical point in the domain. For

cm we have

x =20 cm we have L = 20 2 cm and at the endpoint x = 45 15 5

L = 15 18 6 5 cm.

15 3

cm is the smallest of these values, we conclude that this is the smallest

At the endpoint

Since

possible value for the length of the fold.

w 2 2/3, then the length


of the shortest possible fold is 3 3w/4. Therefore, for a A4 size paper with w = 210 mm and
= 297 mm, the shortest fold is approximately 273 mm; and for a letter size paper with w = 8.5
in and = 11 in, the shortest fold is approximately 11.04 in.

Remark:

If the question is posed for a

sheet of paper with

25

31.

The Rubber Duck is a sculpture designed by Florentijn Hofman and constructed from

PVC. For the purposes of this question, we consider the Rubber Duck to consist of a spherical
head of radius

and a spherical body of radius

b.

The research shows that the cuteness

the Rubber Duck is given by

a
a

b (1 b )(a + b)
K =

if

0 a < b,

if

0 < b a.

Find the dimensions of the cutest Rubber Duck with a total surface area of

Solution:
4b2 =

The total surface area of the Rubber Duck is


400 m2 , giving b2 = 100 a2 . Therefore,

a(100 2a2 )

100 a2
K =

if

4a2 + 4b2 .

400 m2 .

Hence

4a2 +

0 a 5 2,

otherwise,

and we have to nd where the absolute maximum value of

K =
on the interval

0a5 2

100a 2a3
100 a2

occurs.

We rst nd the critical points. As

dK
(100 6a2 )(100 a2 ) (100a 2a3 )(2a)
,
=
da
(100 a2 )2
dK /da = 0

gives

a4 250a2 + 5000 = 0. Now using the quadratic

250 2502 4 5000


2
a =
= 125 25 17 ,
2
26

formula we obtain

of

which gives us four solutions





a = 5 5 + 17, 5 5 17, 5 5 + 17, 5 5 17 .

5
2. On the

a = 5 5 17

Only the rst two of these are positive and the rst one is greater than
other hand,

0 < 5 5 17 < 5 2

as

3<

17 < 5.

We conclude that

is the only critical point.


We have

K = 10 5
for


a = 5 5 17.

As

K =0

17 3
17
>0
17 1

at the endpoints

a=0

and

a=5 2

of the interval,

this is the absolute maximum value.


Therefore the cutest Rubber Duck has

32.


a = 5 5 17

m and

b=5
17 1

m.

A dessert in the shape of a hemisphere with radius 1 dm is made by baking a cylindrical

cake of height

h,

and topping it with a spherical cap of ice cream and surrounding it with a
hemispherical ring of chocolate mousse as shown in the gure. If the cake costs 8/ /dm3 , the
ice cream costs 9/ /dm3 and the chocolate mousse costs 12/ /dm3 , determine the value of

for

(a) the least expensive and (b) the most expensive dessert that can be made.

You may use the fact that the volume of a hemispherical ring of height

h is 2h3 /3.
Ice cream
(9/

/dm3 )

Cake
(8/

/dm3 )

Chocolate
mousse

Solution:

We have

Cost

12
(Volume

of the ring) +

8
(Volume

of the cylinder)

9
(Volume of the cap)

12 2 3 8
9 2
2
=
h + (1 h2 )h + ( 13 h3 (1 h2 )h)
3

3
3
+

= 6 h + 3h3
27

(12/

/dm3 )

and hence we want to nd the absolute maximum and the absolute minimum values
of

= 6 h + 3h3

Cost

We rst nd the critical points. As


gives

h = 1/3

h = 1/3 dm.
the cost is 52/9

dm and

and at that point

for

d(Cost)/dh = 1 + 9h2 , setting this equal to zero


Only the rst of these are in the interval [0, 1],
.

Next we look at the endpoints of the interval.


dm gives Cost

=8

33.

h=1

h=0

dm gives Cost

=6

, and

h=1

Therefore the least expensive dessert has


has

0h1.

h = 1/3 dm, and the most expensive dessert

dm.

We want to build a greenhouse that has a half cylinder roof of radius

and height

mounted horizontally on top of four rectangular walls of height h as shown in the gure. We
have 200 m2 of plastic sheet to be used in the construction of this structure. Find the value
of

for the greenhouse with the largest possible volume we can build.

Solution:

We have

200 m2 = Total

Surface Area

1
1
= 2 (r + 2r)h + 2 r2 + 2r r
2
2
walls

and hence

h=

100
(
r).
3 r

In particular,

half-disks

0 < r 10.

Let

denote the volume of

the greenhouse. Then

V = r 2r h +

1
r2 r
2

bottom
top
and substituting

in terms of
Maximize

in this, our problem becomes:

V =

(400r r3 )
6
28

for

top

0 < r 10.

We rst nd the critical points in the interval

(0, 10].

20
V = (400 3r2 ) = 0 r = m
dr
6
3

Since neither of these satises 0 < r 10 as


3 < 2, there are no critical points
our interval. Now we look at the endpoints. At r = 0 we have lim V = 0, and
r0+
r = 10 we have V = 500 .
Therefore the maximum possible volume is

h=0

34.

Let

500

m3 and occurs when

r = 10

m and

be a continuous function.

x2

Find

f (4)

if

b.

Find

f (4)

if

0
0

f (t) dt = x sin x

f (x)

t2 dt = x sin x

x.

for all

for all

x.
2

x2

x
d
d
Solution: a.
0 f (t) dt = x sin x dx 0 f (t) dt = dx (x sin x)
f (x2 ) 2x = sin x + x cos x. Now letting x = 2 we get f (4) = /2.

gives

Remark:

at

m.

a.

b.

in

f (x)

f (x)

t dt = x sin x
2

f (4) = 0.

t3
] = x sin x
3 0

One might ask if such functions exist. In part

f (x)3
= x sin x.
3

(b), f (x) =

Hence

3x sin x

FTC1

x = 4

is the unique

function satisfying the given condition.


In part

(a),

anything for

sin x

f (x) =
+ cos x for x > 0,
2
2 x
x < 0 so long as it is continuous.

29

and

f (0) =

by continuity; but it can be

35.

Compute

d2 y

dx2 (x,y)=(0,0)

if

x+y

Solution:

satisfying the equation:

we obtain:

is a dierentiable function of

et dt = xy
2

Dierentiating the equation with respect to

x+y

et dt = xy
2

d/dx

x+y
d
d
2
et dt =
xy

dx 0
dx

FTC1

e(x+y) (1 +
2

dy
dy
)=y+x
dx
dx

()

x = 0, y = 0

dy
= 1
dx
Now dierentiating () with respect to

at

(x, y) = (0, 0)

again we get:

e(x+y) (1 +
2

dy
dy
)=y+x
dx
dx

d/dx

d
dy
d
dy
2
(e(x+y) (1 + )) =
(y + x )
dx
dx
dx
dx

e(x+y) (2(x + y))(1 +


2

dy 2 (x+y)2 d2 y dy dy
d2 y
) +e
=
+
+
x
dx
dx2 dx dx
dx2

x = 0, y = 0, dy/dx = 1

d2 y
= 2
dx2

30

at

(x, y) = (0, 0)

36.

Suppose that

is a continuous function satisfying

f (x) = x
for all

x,

and

f (t) dt + x3

f (c) = 1.

is a real number such that

Solution:

Express

f (c)

in terms of

only.

We have:
FTC1

x
x
x
d
d

f (x) = f (t) dt + x f (t) dt + 3x3 = f (t) dt + xf (x) + 3x3


dx
dx 0
0
0
Now substituting

x = c and using the facts that f (c) = c

c
0

f (t) dt + c3 and f (c) = 1

we obtain:

f (c) =

Remark:

c
0

f (t) dt + cf (c) + 3c2 =

It can be shown that

f (x) = 2x(ex

2 /2

f (c) c3
1
+ c + 3c2 = + c + 2c2
c
c

1) is the only function that satises the given

condition.

37.

Evaluate the limit

Solution:

3
sin(xt ) dt
lim 0
x0
x5

We rst make the change of variable

1/3

0 sin(xt ) dt = x
3

t = x1/3 u, dt = x1/3 du,

x4/3

sin(u3 ) du

Now we have:

x4/3

3
x1/3 0 sin(u3 ) du
0 sin(xt ) dt
lim
= lim
x0
x0
x5
x5
x4/3
sin(u3 ) du

= lim 0
x0
x16/3
d x4/3
3
0 sin(u ) du

dx
= lim
x0
16/3 x13/3
L'H

sin(x4 ) 4/3 x1/3


x0
16/3 x13/3
sin(x4 ) 4/3 x1/3
= lim
x0
16/3 x13/3
1
sin(x4 )
= lim
4 x0 x4
1
1
= 1=
4
4

FTC1

= lim

31

to obtain:

38.

Suppose that f is a twice-dierentiable function satisfying f (0) =


8, f (2) = 5, f (2) = 3, f (2) = 7 ; and also suppose that the function

g(x) =
has a critical point at

x = 2.

2, f (0) = 11, f (0) =

1 x
f (t) dt
x 0

Determine whether the critical point of

at

x=2

is a local

minimum, a local maximum or neither.

Solution:

We rst observe that

d
d 1 x
g(x) =
(
f (t) dt)
dx
dx x 0
FTC1

x
x
1
1d
f
(t)
dt
+
f (t) dt

x2 0
x dx 0
x
1
1
= 2 f (t) dt + f (x)
x 0
x

and

as

g (2) = 0

we

must

have

1/4

2
0

f (t) dt + 1/2 f (2) = 0,

and

hence

0 f (t) dt = 2f (2) = 10.


Now dierentiating a second time we obtain that:

x
d
1
1
d
g(x) =
( 2 f (t) dt + f (x))
dx
dx
x 0
x
FTC1

x
x
1 d
1 1
2
f
(t)
dt

f
(t)
dt

+ f (x)

x3 0
x2 dx 0
x2 x
x
2
1
1 1
= 3 f (t) dt 2 f (x) 2 + f (x)
x 0
x
x
x

Substituting

x=2

in this we get

g (2) =
and conclude that

39.

Suppose that

1
1 2
1
3
f (t) dt f (2) + f (2) = < 0

4 0
2
2
2

has a local maximum at

x = 2.

is a continuous and positive function on

[0, 5],

and the area between the

y = f (x) and the x-axis for 0 x 5 is 8. Let A(c) denote the area between the graph
y = f (x) and the x-axis for 0 x c, and let B(c) denote the area between the graph of
dR
= 7, nd
y = f (x) and the x-axis for c x 5. Let R(c) = A(c)/B(c). If R(3) = 1 and
dc c=3
f (3).
graph of
of

Solution:

We have

A(3) = B(3) = 4.

A(3) + B(3) = 8

and

32

R(3) = 1

A(3) = B(3),

implying

As

A(c) =

c
0

f (t) dt

dierentiating these with respect to


Calculus Part 1 we obtain A (c) =
A (3) = f (3) and B (3) = f (3).

and

B(c) =

5
c

f (t) dt ,

c and using the Fundamental Theorem of


f (c) and B (c) = f (c). In particular,

On the other hand,

d
d A(c) A (c)B(c) A(c)B (c)
R(c) =
=
dc
dc B(c)
B(c)2
and letting

7=

Let

gives

d
A (3)B(3) A(3)B (3) f (3)B(3) + A(3)f (3) f (3)
R(c) =
=
=
dc
B(3)2
B(3)2
2
c=3

and hence

40.

c=3

f (3) = 14.

be a continuous function and let

g(x) =
Express

g (x)

f (x)

in terms of

Solution:

For

1 < x < 1

g(x) =

x
1

= x

for

1
1

1 < x < 1.

we rewrite the denition of

f (t) (x t) dt +
x

f (t) x t dt .

f (t) dt

x
1

1
x

g(x)

as follows:

f (t) (t x) dt

f (t)t dt +

1
x

f (t)t dt x

1
x

f (t) dt

Now we dierentiate using the FTC1 to obtain:

g (x) =

=
=

x
1

x
1
x
1

x
x
d
d
f (t) dt
f (t)t dt

dx 1
dx 1
1
1
1
d
d
+
f (t)t dt f (t) dt x
f (t) dt

dx x
dx x
x

f (t) dt + x

f (t) dt + xf (x) f (x)x f (x)x


f (t) dt

Using FTC1 again we compute

1
x

1
x

f (t) dt + xf (x)

f (t) dt

g (x)

to nd

x
1
d
d
f (t) dt
f (t) dt

dx 1
dx x
= f (x) + f (x)
= 2f (x)

g (x) =

for

1 < x < 1.
33

41.

Evaluate the limit

lim (n (

1
1
1
+
++
)) .
2
2
(2n + 1)
(2n + 3)
(4n 1)2

1
on the interval [2, 4]. If we divide this
x2
2
2k
interval into n subintervals of equal length
using the points xk = 2 +
, 0 k n,
n
n
2k 1
and choose our sample points to be the midpoints ck = 2 +
, 1 k n, of these
n

Solution:

Consider the function

f (x) =

subintervals, the Riemann sum for these data becomes

k=1

k=1

f (ck ) xk = f (2 +

n
2k 1 2
2n
) =
n
n k=1 (2n + 2k 1)2

and the denition of the denite integral gives

4 dx
2n
1 4
1 1 1
lim
=
= ] = + = .
2
2
n
x 2
4 2 4
2 x
k=1 (2n + 2k 1)
n

Therefore

lim (n (

42.

1
1
1
1
+
++
)) = .
2
2
2
(2n + 1)
(2n + 3)
(4n 1)
8

Evaluate the following integrals.

a.

2
2
x sin(x ) cos(x ) dx

b.

1
0 x 1 x dx

Solution: a.

Let

u = sin(x2 ).

Then

du = 2x cos(x2 ) dx,

and

1 u2
1
1
2
2
x
sin(x
)
cos(x
)
dx
=
u
du
=

+ C = sin2 (x2 ) + C .

2
2 2
4

b.

Let

u = 1 x.
1

Then

du = dx,

In part

x = 0 u = 1, x = 1 u = 0.

Therefore:

0
1

x 1 x dx = (1 u)u1/2 (du) = (u1/2 u3/2 ) du


1

=[

Remark:

and

u3/2
3/2

1
u5/2

5/2

] =
0

2 2 4
=
3 5 15

(a), if we let u = cos(x2 ), then we obtain the answer


1
2
2
2
2
x sin(x ) cos(x ) dx = 4 cos (x ) + C ;
34

1
sin(2x2 ), and then let u = sin(2x2 ),
2
1
2
2
2
x sin(x ) cos(x ) dx = 8 cos(2x ) + C .

and if we rst observe that


obtain the answer

sin(x2 ) cos(x2 ) =

then we

These are all correct answers. If we write

1
2
2
2
2
x sin(x ) cos(x ) dx = 4 sin (x ) + C1
1
2
2
2
2
x sin(x ) cos(x ) dx = 4 cos (x ) + C2
1
2
2
2
x sin(x ) cos(x ) dx = 8 cos(2x ) + C3
then

43.

C2 = C1 +

1
4

Show that

C3 = C1 +

1
.
8

a
a
f (x)
0 f (x) + f (a x) dx = 2

Solution:
Let

and

Let

u = a x.

for any positive continuous function on

[0, a] .

a
f (x)
I =
dx .
0 f (x) + f (a x)

Then

du = dx, x = 0 u = a, x = a u = 0,

and

a
0
a
f (x)
f (a u)
f (a x)
I =
dx =
(du) =
dx .
0 f (x) + f (a x)
a f (a u) + f (u)
0 f (x) + f (a x)
Therefore

a
f (x)
f (a x)
dx +
dx
0 f (x) + f (a x)
0 f (x) + f (a x)
a f (x) + f (a x)
dx
=
0 f (x) + f (a x)
a

2I =

=
and

I=

a
0

dx = a

a
.
2

(See

the remark on the next page .)

35

Remark:

Here is an explanation of what is going on with no integral signs: Consider the

x- and
y = f (x)/(f (x) + f (a x)) is

rectangle with a vertex at the origin, and sides along the positive
lengths

and 1, respectively.

respect to the center

(a/2, 1/2)

The graph of

y -axes

with

symmetric with

of this rectangle, and therefore divides it into two regions of

equal area. Since the area of the rectangle is

a
I= .
2

the

36

and

is the area of the lower half, we have

44.

Let

y = x x2 and
R about the x-axis.

be the region bounded by the parabola

volume of the solid generated by revolving

the

x-axis,

and let

a.

Express

as an integral using the disk method. (Do not compute! )

b.

Express

as an integral using the cylindrical shell method. (Do not compute! )

Solution:

The parabola intersects the

x-axis at x = 0 and x = 1.

be the

Therefore we have

V = R(x)2 dx = (x x2 )2 dx
0
0
for part

For part

(a).

(b), the cylindrical shell method gives a y-integral,


d

V = 2 (radius
c

of the shell)(height of the shell) dy

as the region is revolved about the

x-axis.

We have

c = 0

and

d = 1/4 ,

the

y-

coordinate of the highest point of the parabola. The radius of the shell is the vertical
distance from the red rectangle in the gure to the

x-axis,

which is

y.

The height

of the shell is the horizontal length of the rectangle; that is, the dierence between
the

x-coordinates

of the right and the left sides of the rectangle.

1
+
1 4y
equation y = xx2 for x, we nd these values as x =
2
respectively. Hence

V = 2

1/4
0

y(

1+

and

1 4y 1 1 4y

) dy .
2
2

37

By solving the

x=

1 4y
,
2

45.

R be the region bounded by the curve y 2 = x2 x4 . Let V be the volume obtained by


rotating R about the x-axis. Let W be the volume obtained by rotating R about the y -axis.
Let

a.

Express

b.

Express

c.

Compute

using both the disk method and the cylindrical shells method.

Solution:

and

W.

By symmetry,

is 2 times the volume obtained by revolving the portion

x-axis. In the disk method, the red


vertical rectangles, when revolved about the x-axis, form the disks that are used in

the computation of V , and therefore the radii of the disks are given by
x2 x4 .
of

using both the disk method and the cylindrical shells method.

lying in the rst quadrant about the

Hence:

V = 2 (radius
0

Again by symmetry,
of

2
of disk) dx

1
= 2 ( x2 x4 )2 dx
0

is 2 times the volume generated by revolving the portion

lying in the rst quadrant about the

y -axis.

This time in the washer method,

the green horizontal rectangles, when revolved about the


that are used in the computation of

y -axis,

form the washers

W.

To nd the outer and the inner radii of the washers we have to solve y 2 =
x2 x4 for x.
2
2
2
2
2
Applying the quadratic formula to (x ) x +y = 0 we obtain x = (1 1 4y 2 )/2,

38

and this gives us

x=

(1 +

1 4y 2 )/2

and

x=

(1

1 4y 2 )/2

as the two nonnegative solutions. Hence:

W = 2
0
= 2

1/2

1/2
0

((outer

2
radius of washer)

1+

((

1 4y 2
) (
2

(inner

2
radius of washer) ) dy
2

1 4y 2
) ) dy
2

Now we consider the cylindrical shells method for both volumes.


about the

y -axis

When revolved

the red vertical rectangles generate the cylindrical shells that are

used in the computation of

by
x2 x4 . Hence:

W = 2 2 (radius
0

W,

and therefore the heights of these shells are given

of shell)(height of shell) dx

When revolved about the

x-axis

1
= 2 2 x x2 x4 dx
0

the green horizontal rectangles generate the

V . Therefore the heights

(1 1 4y 2 )/2. Hence:

cylindrical shells that are used in the computation of

of these shells are given by

V = 2 2
0
= 2 2

1/2

1/2
0

(1 +

(radius

1 4y 2 )/2

of shell)(height of shell) dy

1+

y(

1 4y 2

Finally we compute the volumes. To compute

1 4y 2
) dy
2

we use the integral we obtained

with the disk method,

x3 x5 1 4
V = 2 (x x ) dx = 2 [ ] =
3
5 0 15
0
1

39

and to compute

we use the integral we obtained with the washer method

W = 2
0
= 2
= 2
=

1/2

1/2
0
/2
0
/2

1+

1 4y 2 1

1 4y 2
) dy
2

1 4y 2 dy

cos

1
cos d
2

cos2 d

1 + cos 2
d
2
0
sin 2 /2
= [ +
]
2
4 0
2
=
4
=

/2

where we made the change of variable

Remark:
46.

Compare this example with

y=

1
1
sin , dy = cos d.
2
2

Example 27 in Part 2.

Find the absolute maximum and the absolute minimum values of

interval

[2, 2].

Solution:

f (x) = 2xex + (x2 3)ex = (x2 + 2x 3)ex .


(x2 + 2x 3)ex = 0

Since

x2 + 2x 3 = 0

does not belong to the interval

Hence we are going to compute

f (x) = (x2 3)ex

We want to solve

on the

f (x) = 0.

x = 1, 3 .

[2, 2], the only


x = 1,2, 2.

critical point is

x = 1.

at the points

f (1) = 2e, f (2) = e2 , and f (2) = e2 . Since e > 1 we have e2 > e2 > 2e.
the absolute maximum is e2 and the absolute minimum is 2e.
40

Therefore

47.

Find the absolute maximum and the absolute minimum values of

Solution:

Let

y = x1/x .

Then

ln y =

obtain

ln x
.
x

x1/x .

Dierentiating this with respect to

1
1 dy d
d ln x x x ln x 1 1 ln x
=
ln y =
=
=
y dx dx
dx x
x2
x2

we

1 ln x
d 1/x
x = x1/x
.
dx
x2

Since 1 ln x > 0 for 0 < x < e and 1 ln x < 0 for x > e, the absolute maximum value
of x1/x occurs at x = e and is e1/e . x1/x has no absolute minimum value.

Remark:

Although the reasoning above does not require it, let us also look at what happens

at the endpoints of the domain. Since


L'H

ln x
1/x
lim ln y = lim
= lim
=0,
x
x x
x 1
we have

48.

lim x1/x = lim y = lim eln y = e0 = 1.

Evaluate the limit

Solution:

xk =

lim (

2k/n
k=1

partition

1
n

On the other hand,

lim x1/x = 0

x0+

as

0 = 0.

1 n k/n
2 ).
n k=1
n

is a Riemann sum

f (ck ) xk

for

f (x) = 2x

on

[0, 1]

for the

k=1

k
, 0 k n,
n

and the sample points

ck =

k
, 1 k n.
n

Therefore,

1
2x
1 n
1
lim ( 2k/n ) = 2x dx =
] =
.
n n
ln 2 0 ln 2
0
k=1

Remark:

Alternatively, we can use the formula for the sum of a nite geometric series

2k/n =
k=1

2(n+1)/n 21/n
21/n
=
,
21/n 1
21/n 1

and then

1
1 n k/n
21/n
t2t
2t + t ln 2 2t
=
lim
=
lim
=
lim
.
2 ) = n
t
t
1/n
+
+
t0
n k=1
ln 2 2
ln 2
n(2 1) t0 2 1
L'H

lim (

41

49.

Evaluate the limit

Solution:
form 1 .

Since

sin x 1/x
) .
x0
x
sin x
1
lim
= 1 and lim 2 = ,
x0 x
x0 x

lim (

this is limit has the indeterminate

sin x
2
ln (
)
sin x 1/x
x
Let y = (
) . Then ln y =
. As x 0, this will have the indeterminate
x
x2
0
form
and we can use L'Hpital's Rule.
0
ln(sin x) ln x
lim ln y = lim
x0
x0
x2
cos x 1

x cos x sin x

= lim sin x x = lim


x0
x0
2x
2x2 sin x
cos x x sin x cos x
sin x

= lim
= lim
2
x0 4x sin x + 2x cos x
x0 4 sin x + 2x cos x
sin x

1
1
x
= lim
=
=
x0 sin x
4+2
6
4
+ 2 cos x
x
L'H

L'H

Here applications of L'Hpital's Rule are indicated with

L'H

. Then

sin x 1/x
lim (
)
= lim y = lim eln y = e1/6
x0
x0
x0
x
using the continuity of the exponential function.

Remark:

The rst example of an application of L'Hpital's Rule in Guillaume Franois

Antoine Marquis de L'Hpital's book Analyse des Inniment Petits pour l'Intelligence des

Lignes Courbes of 1696:

There is a typo. Can you nd it?

42

50.

Evaluate

cos(2x) e2x
x0
sin4 x

lim

Solution:
cos(2x) e2x
x4
cos(2x) e2x
lim
=
lim
(

)
x0
x0
x4
sin4 x
sin4 x
2
cos(2x) e2x
x 4
= lim

(lim
)
x0
x0 sin x
x4
2
cos(2x) e2x
= lim
x0
x4
2
2 sin(2x) + 4xe2x

= lim
x0
4x3
2
2
4 cos(2x) + 4e2x 16x2 e2x

= lim
x0
12x2
2
2
8 sin(2x) 48xe2x + 64x3 e2x

= lim
x0
24x
sin(2x)
8
2
2
= lim (
2e2x + x2 e2x )
x0
3x
3
2
4
= 2+0=
3
3
2

L'H

L'H

L'H

Remark:

It is easier to solve this problem using the Taylor series, which will be seen in Calculus

II:

cos(2x) e2x
x0
sin4 x

lim

(2x)2 (2x)4 (2x)6


(2x2 )2 (2x2 )3
+

+ ) (1 + (2x2 ) +
+
+ )
2!
4!
6!
2!
3!
= lim
4
x0
x3
(x
+ )
3!
56
4
x4 + x6 +
45
= lim 3
2
x0
x4 x6 +
3
4 56 2
+ x +
= lim 3 45
2
x0
1 x2 +
3
4
=
3
(1

43

51.

Find the value of the constant

the limit for this value of

Solution:

sin(x + ax3 ) x
x0
x5

a for which the limit lim

a.

exists and compute

We have

sin(x + ax3 ) x
x0
x5
cos(x + ax3 )(1 + 3ax2 ) 1

= lim
x0
5x4
sin(x + ax3 )(1 + 3ax2 )2 + cos(x + ax3 )(6ax)

= lim
x0
20x3
3
cos(x + ax )(1 + 3ax2 )3 sin(x + ax3 ) 3(1 + 3ax2 )(6ax) + cos(x + ax3 )(6a)

.
= lim
x0
60x2

lim
L'H

L'H

L'H

The numerator of the fraction inside this last limit goes to


the limit does not exist unless

a = 1/6.

If

a = 1/6

1+6a as x 0.

Therefore

then

sin(x + x3 /6) x
x0
x5
cos(x + x3 /6)(1 + x2 /2)3 sin(x + x3 /6) 3(1 + x2 /2)x + cos(x + x3 /6)
= lim
x0
60x2

lim

is the sum of

cos(x + x3 /6)(1 (1 + x2 /2)3 )


x0
60x2
cos(x + x3 /6)(3/2 + 3x2 /4 + x4 /8)
1
= lim
=
x0
60
40

lim

and

sin(x + x3 /6) 3(1 + x2 /2)x


x0
60x2
(1 + x2 /2)(1 + x2 /6)
sin(x + x3 /6)
1

lim
= .
= lim
3
x0
x0
x + x /6
20
20

lim

Hence

sin(x + x3 /6) x
3
= .
5
x0
x
40

lim

Remark:

Once again there are shorter ways of doing this. If we use the Taylor series then

(x + ax3 )3 (x + ax3 )5
+

3!
5!
1
1
1
= x + (a ) x3 + (
a) x5 +
6
120 2

sin(x + ax3 ) = (x + ax3 )

and it is immediate that the limit exists exactly when

44

a = 1/6

and then its value is

3/40.

52.

Let b > a > 0 be constants. Find


(x b)2 + y 2 = a2 about the y -axis.

Solution:

the area of the surface generated by revolving the circle

We have

Surface Area

= 2

x
c

dx
1 + ( ) dy
dy

for a surface generated


by revolving a curve about the y -axis. We have x = b +

a2 y 2 and x = b a2 y 2 for the right and left halves of the circle, respectively.
Then

dx
y
=
dy
a2 y 2

dx
a
.
1+( ) =
dy
a2 y 2

Hence

a
dy
a2 y 2 )
2
a
a y2

a
a
+ 2 (b a2 y 2 )
dy
a
a2 y 2
a
1
dy
= 4ab
a
a2 y 2
a

Surface Area

= 2 (b +

y
= 4ab arcsin ( ) ]
a a

= 4ab (arcsin 1 arcsin(1)) = 4ab ( ( )) = 4 2 ab


2
2

Remark:

The surface generated by revolving a circle about a line (in the same plane) that

does not intersect it is called a torus.

53.

if

dy
= xy 2
dx

Solution:

We have

Find

y(1)

dy
= xy 2
dx
Then

y(0) = 1

and

y(0) = 1.

dy
= x dx
y2

1 = C .

Hence

y=

dy
=
x dx
y2

2
2 x2
45

1 1
= x2 + C .
y 2

. This in turn gives

y(1) = 2.

Remark:
54.

What would your answer be if the question asked

y(2)?

Nitrogen dioxide is a reddish-brown gas that contributes to air pollution, and also gives the

smog its color. Under sunlight it decomposes producing other pollutants, one of which is ozone.
As nitrogen dioxide decomposes, its density decreases at a rate proportional to the square of
the density. Suppose that the density of nitrogen dioxide
minutes and

6/25

grams per liter at time

t=3

1/2 grams per liter at time t = 0


Q when t = 15 minutes. (Assume

is

minutes. Find

that no new nitrogen dioxide is added to the environment.)

Solution:
k dt

and

dQ/dt = kQ2 where k is a positive


integrating we obtain 1/Q = kt + C where C
We have

constant. Then

dQ/Q2 =

is a constant.

t = 0 minutes we nd 1/(1/2) = 1/Q(0) = C , hence C = 2 liters per gram.


t = 3 min gives 1/(6/25) = 1/Q(3) = k 3 2 and hence k = 13/18
per gram per minute. Finally we obtain Q = 18/(13t + 36).

Letting

Now letting
liters

Using this we compute the density of nitrogen dioxide as

Q(15) = 6/77

grams per

liter after 15 minutes.

55.

f is a function that has a continuous second derivative and


f (0) = 4 , f (1) = 3 , f (0) = 5 , f (1) = 7, f (0) = 8 and f (1) = 11 . Show that:
Suppose that

that satises

0 f (x)f (x) dx 1

Solution:

du =

We rst do an integration by parts with

f (x) dx, v = f (x), to obtain:

u = f (x), dv = f (x) dx,

hence

2
0 f (x)f (x) dx = [f (x)f (x)] 0 (f (x)) dx

The rst term on the right is equal to f (1)f (1) f (0)f (0) = 3 7
the second term is nonnegative as (f (x))2 0. The result follows.

Remark:

It can be shown that:

0 f (x)f (x) dx < 0

56.

Evaluate the following integrals.

a.

3
sin x sin 2x dx

b.

e ln x
1 dx
x

46

4 5 = 1,

and

Solution: a.

We use the identity

sin 2x = 2 sin cos x

to obtain

3
3
sin x sin 2x dx = sin x 2 sin x cos x dx

= 2 sin4 x cos x dx
= 2 u4 du
2
= u5 + C
5
2
= sin5 x + C
5
after the substitution

u = sin x, du = cos x dx.

b.

u=

The substitution

x, du = dx/(2 x)
e

ln x
dx = 4
1
x

gives:

ln u du

= 4 [u ln u u]

1
= 4 ( e ln e e + 1)

=42 e

Remark:

There are other ways of doing these. Here are some:

Solution: a.
substitution

Use the double-angle formula

u = 1 cos 2x, du = 2 sin 2x dx :

sin2 x = (1 cos 2x)/2

and then the

1 cos 2x
) sin 2x dx
sin x sin 2x dx = (
2
1
= u3/2 du
4 2
1
= u5/2 + C
10 2
1
= (1 cos 2x)5/2 + C
10 2
3

Or you can run this by simply saying

u = sin2 x, du = 2 sin x cos x dx = sin 2x dx

3
3/2
sin x sin 2x dx = u du
2
= u5/2 + C
5
2
= sin5 x + C
5

47

and:

Or use the identity

sin3 x = (3 sin x sin 3x)/4 and the trigonometric product to sum

formulas:

1
3
sin x sin 2x dx = 4 (3 sin x sin 3x) sin 2x dx
1
= (3 sin x sin 2x sin 3x sin 2x) dx
4
1
= (3(cos x cos 3x) (cos x cos 5x)) dx
8
1
= (cos 5x 3 cos 3x + 2 cos x) dx
8
1
1
1
=
sin 5x sin 3x + sin x + C
40
8
4
Or, if you are willing to go complex, use the identity

sin = (ei ei )/(2i) :

eix eix e2ix e2ix


)
dx
sin
x
sin
2x
dx
=
(

2i
2i
1
=
(e5ix + e5ix 3e3ix 3e3ix + 2eix + 2eix ) dx
16
1 e5ix e5ix e3ix e3ix
eix eix
= (

+2
)+C
16
5i
i
i
1
1
1
=
sin 5x sin 3x + sin x + C
40
8
4
3

b.

Do integration by parts:

e ln x
e

1 dx = 1 ln x d(2 x)
x
e dx

e
= [2 x ln x]1 2 x
x
1
e dx

= 2 e 2
1
x

e
= 2 e 4 [ x]1

=2 e4 e+4

=42 e
Or do the other integration by parts rst,

and then from

e 1
ln x
dx = d(x ln x x)
1
x
x
e
1 e x ln x x
x ln x x
] +
dx
=[
2 1
x3/2
x
1
e
1 e ln x
= 1 + dx [ x]1
2 1
x

1 e ln x
= 2 e + dx
2 1
x

1 e ln x
dx = 2 e

2 1
x
48

solve for

e ln x

1 dx = 4 2 e .
x

Or rst do the substitution

x = eu , dx = eu du,

and then do an integration by parts:

1 u
ln x
dx = eu du
0
x
eu

=
=

u eu/2 du

0
1
0

u d(2eu/2 )
1

= [2ueu/2 ]0 2
=

eu/2 du

0
1/2
u/2 1
2e 4[e ]0
1/2
1/2

= 2e

4e

=42 e

57.

+4

Evaluate the following integrals:

a.

b.

c.

dx
(x2 + 1)2

dx

1 x2 dx

Solution: a.

We rst do a change of variable

x = t2 , dx = 2tdt,

dx = 2 tet dt

and then do an integration by parts,

u = t, dv = et dt

du = dt, v = et :

= 2tet 2 et dt
= 2tet 2et + C


= 2 xe x 2e x + C

b.

We use the trigonometric substitution

49

x = sin ,

.
2
2

Then

dx = cos d

and

1 x2 =

1 sin2 = cos2 = cos = cos

as

cos 0

for

.
2
2

1 x2 dx = cos cos d
= cos2 d
1
(1 + cos 2) d
2
1
1
= + sin 2 + C
2
4
1
1
= + sin cos + C
2
2
1
1
= arcsin x + x 1 x2 + C
2
2
=

1
x

1 x2

c.
and

x = tan ,

We use the trigonometric substitution

x2 + 1 = tan2 + 1 = sec2 .

<< .
2
2

Then

dx
sec2
=
(x2 + 1)2 (sec2 )2 d
= cos2 d
=
=
=
=
=

1
(1 + cos 2) d
2
1
1
+ sin 2 + C
2
4
1
1
+ sin cos + C
2
2
1
1
x
1
arctan x +

+C
2
2
x2 + 1
x2 + 1
1
1
x
arctan x + 2
+C
2
2 x +1

x2 + 1

50

dx = sec2 d

Remark:

Other methods can also be used. For instance, the integral in part

(b) can be done

using integration by parts.

58.

Evaluate the following integrals:

a.

1/2

1/2
1/2

b.

1/2

c.

1x
arcsin x dx
1+x

dx

x + 1 x2

dx
1 + ex

Solution: a.
1/2

1/2

We rst re-write the integral as follows:

1/2
1/2
1x
1
x

arcsin x dx +
arcsin x dx
arcsin x dx =
1+x
1/2
1/2
1 x2
1 x2

The rst integral on the right vanishes as the integrand is odd and the integration
interval is symmetric about the origin. We do integration by parts for the second
integral with

u = arcsin x

and

dv =

1 x2 :
1/2

1/2

arcsin x

x
1 x2

1/2

1/2

1/2

c.

Let

dx,

and hence

1 x2 ]

1/2
1/2

du =

1/2
1/2

1 x2

t = sin , dt = cos d.

/6
dx
cos d

=
2
/6 sin + cos
x+ 1x
/6 cos (cos sin )
=
d
/6
cos2 sin2
1 /6 1 + cos 2 sin 2
=
d
2 /6
cos 2
1 /6
=
(sec 2 + 1 tan 2) d
2 /6
/6
1 1
1
= [ ln tan 2 + sec 2 + ln sec 2]
2 2
2
/6

1
= ln( 3 + 2) +
2
6

u = ex + 1, du = ex dx.

Then

ex dx
dx
du
x
=
1 + ex ex + 1 = u = ln u + C = ln(1 + e ) + C
51

and

dx = 1
2 3

1x

arcsin x dx = 1
1+x
2 3

We start by changing variables

1/2

1 x2

dx = [arcsin x

Therefore:

b.

v=

and from this

c dx
dx
=
lim
= lim [ln(1 + ex )]c0 = lim (ln(1 + ec ) ln 2) = ln 2

c
c
1 + ex c 0 1 + ex

follows.

Remark:

Here is another way of doing part

(c).

Let

u = ex , du = ex dx.

Then

dx
ex dx
=
1 + ex ex + (ex )2
du
=
u + u2
1
1
) du
= (
u 1+u
= ln u ln 1 + u + C
= x ln(1 + ex ) + C
and from this

c dx
dx
= lim
x
c 0 1 + ex
1+e
= lim [x ln(1 + ex )]c0
c

= lim (c ln(1 + ec ) + ln 2)
c

= lim ( ln(ec ) ln(1 + ec ) + ln 2)


c

= lim ( ln(ec + 1) + ln 2)
c

= ln 2
follows.

59.

Evaluate the improper integral

0
where

dx
(ax + 1)(x2 + 1)

is a positive constant.

Solution:

We have the partial fraction decomposition

1
a2
ax 1
1
=
(
2
).
2
2
(ax + 1)(x + 1) a + 1 ax + 1 x + 1
Hence:

dx
1
a2
ax 1
=
(
(ax + 1)(x2 + 1) a2 + 1 ax + 1 dx x2 + 1 dx)
=

a2

1
a
(a ln ax + 1 ln(x2 + 1) + arctan(x)) + C .
+1
2

52

Therefore:

c
dx
dx
=
lim

2
(ax + 1)(x + 1) c 0 (ax + 1)(x2 + 1)
c
1
a
= 2
lim [a ln ax + 1 ln(x2 + 1) + arctan(x)]
a + 1 c
2
0

a2

1
a
lim (a ln(ac + 1) ln(c2 + 1) + arctan c
+ 1 c
2
a ln 1 +

=
=
as

60.

ac + 1
lim
=a
c
c2 + 1

The curve

and

a
ac + 1
1

lim
ln
(
)
+
lim arctan c
a2 + 1 c
a2 + 1 c
c2 + 1
a2

.
2

lim arctan c =

is revolved about the

region between the curve and the


solid

1
(a ln a + )
+1
2

y = 1/x, x 1,

a
ln 1 arctan 0)
2

x-axis

for

x1

x-axis

to generate a surface

is revolved about the

x-axis

and the

to generate a

D.

a.

Show that

has nite volume.

b.

Show that

has innite area.

Solution: a.

Using the disk method we obtain


Volume

Since

b.

p = 2 > 1,

R(x)2 dx =

dx
.
x2

this integral converges.

The surface area formula gives

Surface Area

We have

1
x

1+

= 2

1 + (y )2 dx = 2

dx
1 1

0
for x 1, and
= .
x4 x
x
1

1
x

1+

1
dx .
x4

The surface area is innite

by the Direct Comparison Test.

Remark:

We want to get the surface

painted for a reasonable, nite price. We oer the job

to Painter1 and Painter2.

Painter1 says: It cannot be done.

has innite area, it cannot be painted with

nite amount of paint."

Painter2 says: It can be done.

has nite volume. We can ll the inside of

with volume(D ) cubic units of paint and let the excess paint run out."

53

Whom should we believe?

61.

Let

be a nonnegative integer. Show that

Solution:

We use induction on

Let

n = 0.

Let

n>0

Then

tn et dt = n! .

n.

et dt = 1 = 0! .

and assume that

tn1 et dt = (n 1)! .

tn et dt = lim
c

tn et dt

= lim ([tn et ]0 + n

c
lim [tn et ]0
c

= n

Integration by parts gives

+ n

tn1 et dt)
tn1 et dt

tn1 et dt

= n (n 1)!
= n!
where

Remark:

lim cn ec = 0

x > 0.

applications of L'Hpital's Rule.

The Gamma function is dened by

(x) =
for

can be seen after

tx1 et dt

It can be shown that the improper integral on the right converges if and only if

Note that 0 is also a bad point" besides

for

0 < x < 1.

A calculation similar to the one in the solution above shows that


This relation

(x) = (x + 1)/x

(x + 1) = x(x)

for

x > 0.
x > 0.

can be used repeatedly to dene the Gamma function for all

real numbers which are not nonpositive integers.

(n + 1) = n! for all nonnegative integers n, we can use the Gamma function to dene the
factorials of all real numbers which are not negative integers by x! = (x + 1). In particular,

1

1/2 t
u2
( )! = (1/2) = t e dt = 2 e du = 2
= .
2
2
0
0

Since

The volume of an

n=1

(the interval

The case

n=4

n-dimensional ball with radius r is n/2 rn /(n/2)! . Check this formula for
[r, r]), n = 2 (the disk with radius r), and n = 3 (the sphere with radius r).

will be seen in Calculus II.

54

62.

Show that

Solution:

ln x
dx = 0 .
x2 + 1

First of all, by denition,

1 ln x
ln x
ln x
dx =
dx +
dx
2
2
x +1
x2 + 1
0 x +1
1

and the improper integral on the left converges if and only if both basic improper
integrals on the right converge.

Consider

ln x
dx .
x2 + 1
1

Note that

ln x
ln x
2
2
x +1
x

for

x 1.

On the other hand,

ln x
t
dx
=
0 te dt
x2

= lim
c

= lim
c

tet dt

0
c
0

t d(et )
c

= lim ([ tet ]0 +
c

=
=

et dt)

0
t c
lim (ce [e ]0 )
c
lim (cec ec + 1)
c
c

=1
where we used the substitution

x = et , dx = et dt,

followed by the denition of the

improper integral, then an integration by parts and nally the limit


L'H

c
1
=
lim
= 0.
c ec
c ec

lim cec = lim

Therefore by the Direct Comparison Test the improper integral


converges.

Now the convergence of

1 ln x
0 x2 + 1 dx

ln x
dx
x2 + 1

follows as

ln u
1 ln(1/u)
du
ln x
=

dx
=

1 1 + u2 du
(1/u)2 + 1 u2
x2 + 1

where we used the change of variable

x = 1/u, dx = du/u2 ,

and this also gives:

1 ln x
ln x
ln x
dx
=
dx
+
dx

x2 + 1
x2 + 1
0 x2 + 1
1
ln x
ln x
dx
+
dx = 0
=

x2 + 1
x2 + 1
1
1

55

Remark:

Instead of computing

comparison

1
ln x
3/2
2
x
x

for

x1

ln x
dx,
x2

one can show its convergence using the

and the fact that

dx
x3/2

is convergent as

p = 3/2 > 1.

ln x
1

for x 1 can be seen as follows: Consider f (x) =


x ln x on
x2
x3/2

[1, ). Then f (x) = 1/(2 x) 1/x = ( x 2)/(2x)


and x = 4 is the only critical point. Since

f (x) < 0 for x < 4 and f (x) > 0 for x > 4, f (4) = 4 ln 4 = 2 2 ln 2 = 2(1 ln 2) > 0 must be
the absolute minimum value of f on [1, ), and we are done.
Here the fact that

63.*

Determine whether the improper integral

Solution:

ex

dx
ex

converges or diverges.

By denition,

1
dx
dx
=
+

x
x
x
x
x
e e
e ex
0 e e
1

and the given integral converges if and only if both of the integrals on the right
hand side converge.

Let us consider

dx
0 ex ex

rst. Since we have the linearization

ex ex (1 + x) (1 x) = 2x ,
centered at

x = 0,

we expect

1/(ex ex )

to behave like

1/(2x)

near the bad point"

0, and therefore this integral to diverge.


In fact, we have

1
1
1/(ex ex )
x

= lim+ x x = lim+ x x = .
x0 e + e
x0 e e
1/x
2
L'H

L = lim+
x0

1 dx
is divergent (because p = 1 1), we conclude
0 < L < and
x
0
1

dx
dx
diverges
by
the
Limit
Comparison
Test.
Therefore
0 ex ex
0 ex ex

Since
that

diverges too.

Remark:

The other improper integral on the right hand side converges. We have

1
1/(ex ex )
= lim
= 1.
x
x
x
e
1 e2x

L = lim
Since

0<L<
1

and

we conclude that

ex dx = lim ex dx = lim [ex ]c1 = lim (ec + e1 ) = e1 < ,


c 1
c
c
1

ex

dx
ex

converges by the Limit Comparison Test.

*Examples marked red are not part of the Fall 2014 syllabus.
56

64.*

Determine whether the improper integral

Solution:

1 e1/x

dx
x

converges or diverges.

By denition,

1 1 e1/x
1 e1/x
1 e1/x

dx =
dx +
dx
0
1
x
x
x

and the given integral converges if and only if both integrals on the right hand side
converge.
On one hand, since

1 e1/x

x
L = lim+
= lim+ (1 e1/x ) = 1
1
x0
x0

x
is a positive real number, and

1 e1/x

dx
x

1 dx
0
x

converges as

p = 1/2 < 1;

we conclude that

converges by the Limit Comparison Test.

On the other hand, since

1 e1/x

1 e1/x
et
1 et
x
= lim+
L = lim
= lim
= lim+
=1
x
x
1
t0 1
t0
1/x
t
x3/2
L'H

is a positive real number, and

1 e1/x

dx
x

Hence

dx
x3/2

converges as

p = 3/2 > 1;

also converges by the Limit Comparison Test.

1 e1/x

dx
x

converges.

*Examples marked red are not part of the Fall 2014 syllabus.

57

we conclude that

Part 2: Multi-variable Functions

1.

Consider the point

P (3, 5, 1)

L x = 2t 1 , y = t + 2 , z = 2t ; < t < .

and the line

a.

Find the equation of the plane passing through

perpendicular to

b.

Find the equation of the plane passing through

and containing

Solution: a.

L.

L.

v = 2i j 2k . Therefore we can
take the normal vector of the plane to be n = 2i j 2k . Then the equation of the
plane is 2 (x 3) + (1) (y (5)) + (2) (z 1) = 0 , or 2x y 2z = 9 .

b.

The line

is parallel to the vector

n to the plane containing the line L and the point P will be


#
v, and it will also be perpendicular to P Q where Q is any point
on the line. We can take Q(1, 2, 0), the point corresponding to t = 0 , and then
#
P Q = 4i + 7j k . Now we can take the normal vector v to be
A normal

perpendicular to

#
n = v PQ =
or in fact

n = 3i + 2j + 2k .

i
j k
2 1 2 = 15i + 10j + 10k ,
4 7 1

Then the equation of the plane is

3 (x 3) + 2 (y (5)) + 2 (z 1) = 0 ,
or

2.

3x + 2y + 2z = 1 .

Consider the plane

P 3x 4y + z = 10 ,

and the points

a.

Find the equation of the line passing through

b.

Find the equation of the plane passing through

Solution: a.

Since

n = 3i 4j + k

P (2, 3, 1)

perpendicular to

and

b.

P.
P.

is normal to the plane, it will be parallel to any

v = 3i 4j + k ,
x = 3t + 2, y = 4t + 3, z = t 1; < t < .

A plane perpendicular to

Q(1, 2, 2) .

perpendicular to

line perpendicular to the plane. Hence we can take


of the line is

and

will have a normal

and the equation

perpendicular to the normal

n = 3i 4j + k of P . Also a plane containing the points P and Q will have a normal


#
n perpendicular to P Q = i j + 3k . Therefore we can take n to be
#
n = n PQ =

i
j
3 4
1 1
58

k
1 = 11i 10j 7k ,
3

or rather

n = 11i + 10j + 7k .

This gives the equation of the plane as

11 (x 2) + 10 (y 3) + 7 (z (1)) = 0 ,
or

3.

11x + 10y + 7z = 45 .

Find a parametric equation of the line

that intersects both of the lines

L1 x = 2t 1,

y = t + 2,

z = 3t + 1

L2 x = s + 5,

y = 2s + 3,

z = s

and

perpendicularly.

Solution: v1 = 2i j + 3k and v2 = i + 2j k are the velocity vectors of the lines L1


and

L2 ,

respectively. Hence,

i
j k
v = 2 1 3 = 5i + 5j + 5k
1 2 1
is a velocity vector for

L.

So we may take

v = i j k.

Then

i
j k
n = vv1 = 1 1 1 = 4i 5j + k
2 1 3
is normal to the plane
an equation of

containing the lines

and

L1 .

As

P1 (1, 2, 1)

is in

P,

is

4 (x (1)) + (5) (y 2) + 1 (z 1) = 0 ,
or

4x + 5y z = 5.

At the point

P0

of intersection of

and

L2 , s

satises:

4 (s + 5) + 5 (2s + 3) (s) = 5
Hence

s = 2.

Substituting this back in the equations of

Therefore an equation of

x = t + 3,

L2

gives

P0 (3, 1, 2).

is:

y = t 1 ,

z = t + 2 ;

59

( < t < ) .

4.

c be a constant. Show the angle between the position and the velocity vectors along the
r = ect cos t i + ect sin t j , < t < , is constant.

Let

curve

Solution:

We have

r = ect cos t i + ect sin t j


and

v=

dr
= (c ect cos t ect sin t)i + (c ect sin t + ect cos t)j .
dt

Then

r = ((ect cos t)2 + (ect sin t)2 )1/2 = ect ,

v = ((c ect cos t ect sin t)2 + (c ect sin t + ect cos t)2 )1/2 = c2 + 1 ect ,
r v = ect cos t (c ect cos t ect sin t) + ect sin t (c ect sin t + ect cos t) = c e2t .
Therefore, if

is the angle between

cos =
and we conclude that

and

v,

we have

rv
c e2t
c

=
=
,
ct
2
ct
2
r v e c + 1 e
c +1

is constant.

60

5. a.

Show that

b.

Show that

xy 2
=0.
(x,y)(0,0) x6 + y 2
xy
lim
does
6
(x,y)(0,0) x + y 2
lim

Solution: a.

We have

0 y 2 x6 + y 2

0
for all

(x, y) =/ (0, 0) .

not exist.

for all

(x, y).

Hence

xy 2
y2

=
x

x 1 = x
x6 + y 2
x6 + y 2

Since

lim

(x,y)(0,0)

x = 0 ,

the Sandwich Theorem gives

xy 2
=0.
(x,y)(0,0) x6 + y 2
lim

b.

The limit along the

x-axis

lim

(x,y)(0,0)
x-axis

along the

whereas the limit along the

lim

xy
x0
= lim 6
= lim 0 = 0 ,
2
x0 x + 02
x0
+y

x6

y=x

(x,y)(0,0)
y=x

along the line

is

line is

xy
xx
1
= lim 6
= lim 4
=1.
2
2
x0
x0
+y
x +x
x +1

x6

Since these two limits are dierent, the two-variable limit

lim

xy
+ y2

(x,y)(0,0) x6

exist by the Two-Path Test.

Remark:

Let

a, b, c, d

be positive constants. It can be shown that the limit

xa yb
(x,y)(0,0) xc + yd
lim

exists if

6.

a b
+ >1,
c d

and does not exist otherwise.

Determine all values of the constant

>0

for which the limit

x2 y 3
(x,y)(0,0) x3 + y
lim

exists.

Solution:

We observe that

lim

(x,y)(0,0)
x = y3

along the curve

(y 3 )2 y 3
y
x2 y 3
1
=
lim
= lim lim
.
3

3
3

y0 y + y
y0 y y0 1 + y9
x + y
61

does not

1
y0 1 + y9

lim

is

if

>9

and

exist. Therefore the limit of

1/2

= 9.

if

x2 y 3
x3 + y

consequently the two-variable limit

along the curve

x2 y 3
(x,y)(0,0) x3 + y
lim

Now we will show that the limit is 0 if

On the other hand

< 9.

x = y3

y
y0 y

lim

does not

does not exist, and

does not exist either for

9.

We have

x2 y 3
(x/y/3 )2

=
y3/3 y3/3
x3 + y
(x/y/3 )3 + 1

Here we used the fact that if t 1, then t2 t3 and hence


t2

1
, and if 0 < t < 1, then t2 < 1 gives
1. Since
lim y3/3 = 0 for
(x,y)(0,0)
t3 + 1
t3 + 1
x2 y 3
< 9,
lim
= 0 follows by the Sandwich Theorem.
(x,y)(0,0) x3 + y

for all
t2

7.

(x, y) =/ (0, 0).

Determine all values of the positive constant

lim

(x,y)(0,0) (x2

for which the limit

x
+ y 2 )k

exists.

Solution:
Suppose

The limit exists for

k<

1
2

. We have

k<

1
2

, and does not exist for

0 x2 x2 + y 2

for all

(x, y),

1
2

and hence

x
x2
12k
0 2

x
(
) x12k
(x + y 2 )k
x2 + y 2
for all

(x, y) =/ (0, 0) .

1 2k > 0,

Since

Therefore

lim

(x,y)(0,0)

(x2

we have

x12k 0

as

(x, y) (0, 0).

x
=0
+ y 2 )k

by the Sandwich Theorem.

Suppose

k=

1
2

. Then

lim

(x,y)(0,0)
x-axis

along the

and

lim

(x,y)(0,0)
y -axis

along the

(x2

x
x
= lim 2
= lim 1 = 1 ,
2
1/2
x0 (x + 02 )1/2
x0
+y )

0
x
=
lim
= lim 0 = 0 .
(x2 + y 2 )1/2 y0 (02 + y 2 )1/2 x0

62

Since these limits are dierent, the two-variable limit

lim

(x,y)(0,0) (x2

x
+ y 2 )1/2

does not

exist by the Two-Path Test.

k > 1/2 .

Suppose

Then

lim

(x,y)(0,0)
x-axis

along the

(x2

x
x
= lim 2
= lim x12k =
2
k
2
k
x0
x0
+y )
(x + 0 )

does not exist. Therefore the two-variable limit

x
(x,y)(0,0) (x2 + y 2 )k
lim

does not exist

either.

8.

Let

where

xa y b

x4 + y 6
f (x, y) =

0
a

and

exist values of

a.
as

f (x, y)

if

(x, y) =/ (0, 0)

if

(x, y) = (0, 0)

are nonnegative positive integers.

and

for which

is continuous at

limit

(a-e),

determine whether there

(0, 0).

b. f (x, y) goes to 1 as (x, y) approaches (0, 0)


(x, y) approaches (0, 0) along the line y = x.
c.

In each of

satises the given condition.

f (x, y) goes to 0 as (x, y) approaches (0, 0)


lim f (x, y) does not exist.

along the line

y = x,

and

goes to

along any line through the origin, and the

(x,y)(0,0)

d.

f (x, y) goes to 0 as (x, y) approaches (0, 0) along any line through


y -axis, and f (x, y) goes to 1 as (x, y) approaches (0, 0) along the y -axis.

e.

f (x, y)

fx (0, 0)

and

fy (0, 0)

Solution: a.

If

exist, and

a=4

and

f (x, y)

b = 1,

then

is not dierentiable at

is continuous at

the origin except the

(0, 0).

(0, 0).

This follows from

the Sandwich Theorem as

0 f (x, y) =
for

(x, y) =/ 0

b.

Let

a=3

x4
x4 y

y 1 y y
x4 + y 6
x4 + y 6

implies that the limit of


and

b = 1.

lim

f (x, y)

at

(0, 0)

is

0 = f (0, 0).

Then

(x,y)(0,0)
y=x

x4
1
= lim
=1
4
6
x0 x + x
x0 1 + x2

f (x, y) = lim f (x, x) = lim


x0

along the line

and

lim

(x,y)(0,0)
y=x

1
x4
=
lim
= 1 .
x0 1 + x2
x0 x4 + x6

f (x, y) = lim f (x, x) = lim


x0

along the line

63

c.

Let

a=2

and

b = 3.

lim

(x,y)(0,0)
y=mx

Then we have

m3 x5
m3 x
=
lim
=0
x0 1 + m6 x2
x0 x4 + m6 x6

f (x, y) = lim f (x, mx) = lim


x0

along the line

as well as

lim

(x,y)(0,0)
along the y -axis

f (x, y) = lim f (0, y) = lim 0 = 0 .


y0

y0

However,

lim

(x,y)(0,0)
y=x2/3

1 1
x4
=
lim
= =/ 0
x0 2
x0 x4 + x4
2

f (x, y) = lim f (x, x2/3 ) = lim


x0

along the curve

and hence the limit of

d.

Let

a=0

and

lim

f (x, y)

b = 6.

(x,y)(0,0)
y=mx

at

(0, 0)

does not exist by the 2-Path Test.

Then we have

m6 x6
m6 x2
=
lim
=0
x0 1 + m6 x2
x0 x4 + m6 x6

f (x, y) = lim f (x, mx) = lim


x0

along the line

and

lim

(x,y)(0,0)
y -axis

y6
= lim 1 = 1 .
y0 y 6
y0

f (x, y) = lim f (0, y) = lim


y0

along the

e.

a = 1 and b = 1, then f (x, y) is dierentiable at (0, 0) as it is not even continuous


there. This can be seen by considering its limit along the line y = x which does not
exist. On the other hand, fx (0, 0) and fy (0, 0) are both 0 as f is identically zero on
If

both axes.

Remark:

The complete lists of ordered pairs

(a, b)

of nonnegative integers that satisfy the

given conditions are as follows:

a.

all

b.

(3, 1), (1, 3)

c.

(1, 4), (2, 3)

d.

(0, 6)

e.

(0, 7), (1, 1), (1, 2), (1, 3), (1, 4), (1, 5), (2, 1), (2, 2), (2, 3), (2, 4), (3, 1), (3, 2), (4, 1)

(a, b)

with

3a + 2b > 12

64

9.

Assume that

yz 2 + z 3 = 1

and

Solution:
z

and

and

are dierentiable functions of

zw3 xz 3 + y 2 w = 1 .

z
x

Find

at

and

x,

but

satisfying the equations

xw3 +

(x, y, z, w) = (1, 1, 1, 1) .

Dierentiating the equations with respect to

depend on

x and keeping in mind that

does not; we obtain

w3 + x 3w2 wx + y 2zzx + 3z 2 zx = 0 ,
and

zx w3 + z 3w2 wx z 3 x 3z 2 zx + y 2 wx = 0 .
Substituting
Solving for

x = 1, y = 1, z = 1, w = 1,

zx

Let

Suppose

z = f (x, y)

5zx + 3wx = 1

and

2zx + 2wx = 1 .

we nd

z
5
=
x
4

10.

we get

at

(x, y, z, w) = (1, 1, 1, 1) .

be a dierentiable function such that

f (3, 3) = 1,

fx (3, 3) = 2,

fy (3, 3) = 11,

f (2, 5) = 1,

fx (2, 5) = 7,

fy (2, 5) = 3.

is a dierentiable function of

and

satisfying the equation

f (w, w) = f (uv, u2 + v 2 )
for all

(u, v).

Find

Solution:

w
u

at

(u, v, w) = (1, 2, 3).

Dierentiating the identity

f (w, w) = f (uv, u2 + v 2 )
with respect to

fx (w, w)

gives

w
w
(uv)
(u2 + v 2 )
+ fy (w, w)
= fx (uv, u2 + v 2 )
+ fy (uv, u2 + v 2 )
u
u
u
u

by the Chain Rule. Hence

(fx (w, w) + fy (w, w))

w
= fx (uv, u2 + v 2 ) v + fy (uv, u2 + v 2 ) 2u
u

which leads to

w
= 2fx (2, 5) + 2fy (2, 5)
u
after substituting (u, v, w) = (1, 2, 3).
Now using fx (3, 3) = 2, fy (3, 3) = 11,
fx (2, 5) = 7, and fy (2, 5) = 3, we conclude that
(fx (3, 3) + fy (3, 3))

w 8
=
u 9

at

(u, v, w) = (1, 2, 3) .

65

11.

Let

u = x + y + z , v = xy + yz + zx, w = xyz , and suppose that f (u, v, w) is


f (u, v, w) = x4 + y 4 + z 4 for all (x, y, z). Find fu (2, 1, 2).

a dierentiable

function satisfying

Solution:

We can take

Dierentiating

(x, y, z) = (1, 1, 2)

as this gives

(u, v, w) = (2, 1, 2).

f (u, v, w) = x4 +y 4 +z 4 with respect to x, y , z , respectively, we obtain:


fu ux + fv vx + fw wx = 4x3
fu uy + fv vy + fw wy = 4y 3
fu uz + fv vz + fw wz = 4z 3

Now using

u = x + y + z , v = xy + yz + zx, w = xyz ,

these give:

fu 1 + fv (y + z) + fw yz = 4x3
fu 1 + fv (x + z) + fw xz = 4y 3
fu 1 + fv (x + y) + fw xy = 4z 3
Substituting

(x, y, z) = (1, 1, 2)

we get:

fu + fv 2fw = 4
fu + 3fv + 2fw = 4
fu fw = 32
2fu + 8fw = 16, and
6fu = 240. So fu (2, 1, 2) = 40.

Subtracting 3 times the rst equation from the second gives


adding 8 times the third equation to this gives

Remark:
for a given

f (u, v, w) = u4 4u2 + 2v 2 + 4uw is one. Also note that


corresponding (x, y, z) must be the roots of T 3 uT 2 + vT w = 0 and

There are such

(u, v, w),

the

f.

In fact,

hence is determined up to a permutation of its entries, making the answer independent of the
choice.

12.

Let

z = f (x, y)

be a twice-dierentiable function and

x = r cos , y = r sin .

Show that

2f 2f 2z 1 z 1 2z
+
=
+
+
.
x2 y 2 r2 r r r2 2

Solution:

If

z = F (x, y)

y,

then by the chain

z F x F y
=

= Fx cos + Fy sin ,
r x r y r

(i )

z F x F y
=

= Fx (r sin ) + Fy (r cos ) .
x y

(ii )

rule we have

is a dierentiable function of

and

and similarly,

We use

(i ) with F

=f

to obtain

z
= fx cos + fy sin .
r
66

A)

Then

To compute

2z

= (fx ) cos + (fy ) sin .


2
r
r
r

(fx ) and
(fy ) we use (i ) with F = fx and F = fy ,
r
r

respectively:

2z
= (fxx cos + fxy sin ) cos + (fyx cos + fyy sin ) sin
r2
= fxx cos2 + 2fxy cos sin + fyy sin2
Similarly, using

(ii ) with F

=f

B)

gives

z
= fx (r sin ) + fy (r cos ) ,

and dierentiating this with respect to

again gives

2z

= (fx ) (r sin ) + fx (r sin )


2

+ (fy ) (r cos ) + fy (r cos )

= (fxx (r sin ) + fxy r cos )(r sin ) + fx (r cos )


+ (fyx (r sin ) + fyy r cos )(r cos ) + fy (r sin )
2
2
= fxx r sin 2fxy r2 cos sin + fyy r2 cos2
r(fx cos + fy sin )

13.

where we used

(ii ) with F

Now if we add

(B), 1/r times (A), and 1/r2 times (C), we obtain fxx + fyy .

Suppose that

= fx

(x, y) =/ (0, 0).


5, fyy (3, 1) = 4.

Solution:

C)

F = fy .

f (x, y) is a twice-dierentiable function with continuous derivatives satisfying


f(

for all

and

Find

x2

x
y
, 2
) = f (x, y)
2
+ y x + y2

fxx (3/10, 1/10)

if

fx (3, 1) = 8, fy (3, 1) = 7, fxx (3, 1) = 2, fxy (3, 1) =

We have:

fx (x, y) =

x
, 2
)
f( 2
2
x x + y x + y 2

= fx (

x2

x
y

x
, 2
)
)
( 2
2
2
+y x +y
x x + y 2
+ fy (

= fx (

x
y

y
,
)

)
(
x2 + y 2 x2 + y 2 x x2 + y 2

y
1 (x2 + y 2 ) x 2x
x
,
)

x2 + y 2 x2 + y 2
(x2 + y 2 )2
+ fy (

x2

67

x
y
2xy
, 2
) 2
2
2
+y x +y
(x + y 2 )2

Dierentiating this a second time we obtain:

fxx (x, y) =

x
y
y 2 x2

(fx ( 2
,
)

x
x + y 2 x2 + y 2 (x2 + y 2 )2
+ fy (

x2

y
2xy
x
, 2
) 2
)
2
2
+y x +y
(x + y 2 )2

x
y
y 2 x2
= fxx ( 2
,
)

(
)
x + y 2 x2 + y 2
(x2 + y 2 )2
+ fxy (

y
2xy
y 2 x2
x
,
)

x2 + y 2 x2 + y 2 (x2 + y 2 )2 (x2 + y 2 )2

+ fx (

x
y

y 2 x2
,
)

(
)
x2 + y 2 x2 + y 2 x (x2 + y 2 )2
+ fyx (

x
y
y 2 x2
2xy
,
)

2
2
2
2
2
2
2
2
x +y x +y
(x + y ) (x + y 2 )2

x
y
2xy
+ fyy ( 2
, 2
)( 2
)
2
2
x +y x +y
(x + y 2 )2
+ fy (
= fxx (

x
y

2xy
,
)

(
)
x2 + y 2 x2 + y 2 x (x2 + y 2 )2

x
y
(y 2 x2 )2
,
)

(
)
x2 + y 2 x2 + y 2
(x2 + y 2 )4
+ 2fxy (

x
y
y 2 x2
2xy
,
)

2
2
2
2
2
2
2
2
x +y x +y
(x + y ) (x + y 2 )2
2

x
y
2xy
+ fyy ( 2
, 2
)( 2
)
2
2
x +y x +y
(x + y 2 )2
+ fx (

x
y
2x(x2 3y 2 )
,
)

x2 + y 2 x2 + y 2
(x2 + y 2 )3
+ fy (

Now letting

(x, y) = (3/10, 1/10)

x
y
2y(3x2 y 2 )
,
)

x2 + y 2 x2 + y 2
(x2 + y 2 )3

gives:

fxx (3/10, 1/10) = fxx (3, 10) (8)2 + 2fxy (3, 10) (8) (6) + fyy (3, 10) (6)2
+ fx (3, 10) 36 + fy (3, 10) 52
= 2 64 + 2 5 48 + (4) 36 + (8) 36 + 7 52
= 540

68

14.

Let

P0 (3, 2)

f (x, y) = x3 y xy 2 + cx2

c is a constant.
A = 2i + 5j.

where

in the direction of the vector

Find

c if f

increases fastest at the point

Solution:

f = (3x2 y y 2 +2cx)i+(x3 2xy)j (f )P0 = (50+6c)i+15j . Since f


increases the fastest at P0 in the direction of (f )P0 , A = 2i + 5j must be a positive
c = 22/3 . Finally we check that
multiple of (f )P0 . Hence (50 + 6c)/2 = 15/5
c = 22/3 gives (f )P0 = 6i + 15j = 3A which is indeed a positive multiple of A.

15.

Find a vector that is tangent to the intersection curve of the surfaces

z = xy

at the point

Solution:
are level

P0 (1, 2, 2).

f (x, y, z) = x2 +y 2 +z 2
surfaces of f and g .
Let

and

g(x, y, z) = xy z .

x2 + y 2 + z 2 = 9

Then the given surfaces

f = 2xi + 2yj + 2zk (f )P0 = 2i + 4j + 4k and g = yi + xj k


= 2i + j k . (f )P0 is normal to the surface dened by x2 + y 2 + z 2 = 9 and
is normal to the surface dened by z = xy . Therefore,

We have

(g)P0
(g)P0

(f )P0 (g)P0 =

i
2
2

j k
4 4
1 1

= 8i + 10j 6k ,

or any multiple of it, is tangent to both of these surfaces, and hence, to their curve
of intersection at

16.

Let

P0 .

be a constant. Find and classify all critical points of

Solution:

f (x, y) = x3 3axy + y 3 .

fx = 3x2 3ay = 0 and fy = 3ax + 3y 2 = 0. If a = 0,


=0x=0
= 0 y = 0, and (0, 0) is the only critical point.
If a =
/ 0, then the rst equation gives y = x2 /a, and substituting this in the second
equation we get x4 a3 x = 0 whose solutions are x = 0 and x = a. Now using y = x2 /a,
we get (0, 0) and (a, a) as the critical points.
At a critical point

then 3x2

and 3y 2

We compute the discriminant:

fxx (a, a) = 6a, (a, a) is a local minimum for a > 0 and


local maximum for a < 0. On the other hand, (0, 0) = 9a2 implies that (0, 0) is
saddle point for a =
/ 0.

As

(a, a) = 27a2

6x 3a
fxx fxy
=

fyz fyy
3a 6y

and

Now we look at the sole critical point

(0, 0)

a
a

a = 0. As (0, 0) = 0,
restrict f (x, y) = x3 + y 3 to the

in the case

the second derivative test fails in this case. If we


we get f (x, 0) = x3 . Since this single variable function does not have a local

x-axis

x = 0, f (x, y) cannot have a local maximum


conclude that (0, 0) is a saddle point when a = 0.

maximum or minimum at
at

(0, 0)

either. We

69

or minimum

and

17.

Find the absolute maximum and minimum values of the function


on the unit disk D = {(x, y) x2 + y 2 1}.

f (x, y) = 2x3 +2xy 2 xy 2

Solution:

We rst nd the critical points of f (x, y) in the interior of D . At a


critical point we have fx = 6x2 + 2y 2 1 = 0 and fy = 4xy 2y = 0. The second
equation implies that
obtain

y = 1/ 6

y=0

or

x = 1/2.

Substituting these into the rst equation we

in the rst case, and no solution in the second case. Therefore

(x, y) = (1/ 6, 0)
interior of D .

the critical points are


points lie in the

and

(1/ 6, 0).

Note that both of these

D, that is, the unit circle x2 + y 2 = 1. We can


1. These solutions correspond to the upper
and lower semicircles.
Then f (x, 1 x2 ) = x2 + x 1 for 1 x 1, and

d
1
f (x, 1 x2 ) = 2x + 1 = 0 x = . This gives the critical points (x, y) =
dx
2

(1/2, 3/2) and (1/2, 3/2) of the restriction of f to the boundary of D. We


must also include the endpoints x = 1 and x = 1, in other words, the points
(x, y) = (1, 0) and (1, 0) in our list.
Now we look at the boundary of

solve y as y = 1 x2 , 1 x

Hence the absolute maximum and the absolute minimum of

on

occur at some

of the points

(1/ 6, 0), (1/ 6, 0), (1/2, 3/2), (1/2, 3/2), (1, 0), (1, 0) .

The values of

at these points are

2 1
,
3 3

2
5
5
, , , 1 , 1 ,
3
4
4

respectively. Therefore the absolute maximum value is 1 and the absolute minimum
value is

Remark:

5/4.

Here are two more ways of dealing with the critical points of the restriction of

the boundary of

D.
70

to

In the rst one we parametrize the boundary, which is the unit circle, by

< t < .

x = cos t, y = sin t,

Then

d
d
1
f (cos t, sin t) = (cos2 t + cos t 1) = 2 cos t sin t sin t = 0 cos t =
dt
dt
2

and these give us (x, y) = (1/2,


3/2), (1/2, 3/2), (1, 0), (1, 0).

or

sin t = 0 ,

g(x,y)

x2 + y 2 = 1.

In the second we use the Lagrange Multipliers Method for the boundary

f = g
}
g = 1

O2

f = gx

fy = gy

g = 1

6x2 + 2y 2 1 = 2x

4xy 2y = 2y

x2 + y 2 = 1

O1
O2
O3

gives y = 0 or x = (1 +)/2. If y = 0, then 3 gives x = 1 (and = 5/2). On the other hand,

2 2x + 1 = 0 x = 1/2 and y = 3/2. Therefore


x = (1 + )/2, then from 1 and 3 , 4x
the points (x, y) = (1/2,
3/2), (1/2, 3/2), (1, 0), (1, 0) are added to the list.

if

18.

1, x and y , where 1 x y 0, are stacked on top of each


the largest possible value of the total height h.

Three hemispheres with radiuses

other as shown in the gure. Find

Solution:

We want to maximize

h(x, y) =

1 x2 +

x2 y 2 + y

on the closed and bounded region

D = {(x, y) 0 y x 1}.

We rst nd the critical points of

hx =

in the interior of

x
+
1 x2
x2 y 2
x

and

hy =

Setting the second equation equal to zero we obtain

x=

2y

where we used the fact that


rst equation gives 1 2y 2 = 2y 2 y 2

x = 2/3. So
the only critical

as 0 1/ 3
2/3 1.

point

D.

We have

y
x2 y 2

+ 1.

y 2 = x2 y 2

x2 = 2y 2

x > 0 and y > 0. Now substituting


this in the

2
3y = 1
y = 1/
3 as y > 0 and hence
of h is (x, y) = ( 2/3, 1/ 3) and it lies in D

71

Now we consider the restriction of

to the boundary of

D.

y =0
1 x2 + x

Side 1 : On the bottom edge of the triangle, we


have

therefore we are considering the function

x
d
h(x, 0) =
+1=0
dx
1 x2
x > 0. Taking
the endpoints x = 0
points (x, y) = (1/ 2, 0), (0, 0), (1, 0).
as

h(x, 0) =

x2 = 1 x2
and

x=1

0 x 1,
0 x 1.

and
for

2x2 = 1

and

1
x=
2

into account, Side 1 gives us the

Side 2 : On the right edge of the triangle,


we have x = 1 and
h(1, y) = 1 y 2 + y for 0

0 y 1, and therefore
y
1. As in the case of

(x, y) = (1, 1/ 2), (1, 0), (1, 1).

we are considering the function

Side 1 this leads to the points

Side 3 : On the top edge of the triangle, we


have y = x and 0 x 1, and therefore
we are considering the function h(x, x) =
1 x2 + x for 0 x 1. Once again as in
the case of Side 1 we obtain the points

Now we nd the values of

(x, y) = (1/ 2, 1/ 2), (0, 0), (1, 1).

at these seven points:

f (0, 0) = f (1, 0) = f (1, 1) = 1

f (1/ 2, 0) = f (1, 1/ 2) = f (1/ 2, 1/ 2) = 2

f ( 2/3, 1/ 3) = 3
Therefore the maximum possible total height of the three hemispheres is

Remark:

3.

Here is a single variable argument which solves the problem for any number

k for some k 1.
Then for k + 1 hemispheres, where the second one from the bottom has radius r , the maximum

possible height will be H(r) =


kr + 1 r2 for 0 r 1. H is 0 at r = 0 and r = 1 and its only

k/(k + 1) where its value is k +1. Therefore we conclude inductively


critical point is at r =
n for n 1.
that the maximum possible height for n hemispheres is
hemispheres: Suppose the maximum possible height for

72

hemispheres is

19.

Find the absolute maximum and the absolute minimum of


sphere x2 + y 2 + z 2 = 1.

Solution:
z 2 1.

We will use the Lagrange Multipliers Method. Let

O2

If

and

z = 0,

O3

we obtain

then

the points

O3

gives

(1, 0, 0).

fx
fy
fz
g

=
=
=
=

z = 42 z ,
y = 0,

gx
gy
gz
0

3x2

2 + y2 + z2

and hence

and then

O4

z=0
gives

or

= 1/2

x = 1.

O O
O
O
O1 O3 3x = x y= z

on the unit

g(x, y, z) = x2 + y 2 +

Then

f = g
}
g = 0

From

f (x, y, z) = x3 + yz

O1
O2
O3
O4

=
=
=
=

2x
2y
2z
1

or

= 1/2.

In this case we have

= 1/2, then from 1 and 3 , 3x2 = x and y = z . Therefore we either


have x = 0 in which case y = z = 1/ 2 by 4 , or we have x = 1/3 and then

y = z = 2/3 again by 4 . In this case the critical points are (0, 1/ 2, 1/ 2)


and (1/3, 2/3, 2/3).
If

If

= 1/2,

then from

and

and

. A reasoning similar to

(0, 1/ 2, 1/ 2) and (1/3, 2/3, 2/3).

Hence the critical points are (1, 0, 0), (0, 1/ 2, 1/ 2), (1/3, 2/3, 2/3),

(0, 1/ 2, 1/ 2), (1/3, 2/3, 2/3) , and the values of f at these points are
1, 1/2, 1/2, 13/27, 13/27 , respectively. Therefore the absolute maximum is 1 and
the absolute minimum is 1.
the previous case gives the points

73

20.

Evaluate the following integrals:

a.

3
0 y sin(x ) dx dy

b.

2 x
R y e dA
2

2yy 2

c.

0 0

d.

where

R = {(x, y) 0 y x}

xy
dx dy
+ y2

x2

(x2

dy dx
+ y 2 )2 + 1

Solution: a.

We will rst express the iterated integral as a double integral and

x-integral goes from x = y to x = 1 as


gure in the xy -plane. Then the y -integral

then reverse the order of integration. The


shown by the red line segments in the
goes from

y=0

to

y = 1.

Therefore the intervals of the x-integral trace out the region R bounded by the
parabola y = x2 , the line x = 1, and the x-axis. Note that the x-integrals are always
from left to right in the interval

0 y 1.

Hence we have

3
3
0 y sin(x ) dx dy = R sin(x ) dA .

y -integral rst.
integration for the y -

Now we express this double integral as an iterated integral with the


The green line segment in the gure shows the interval of
integral which goes from y = 0 to y = x2 . Then:

74

3
3
0 y sin(x ) dx dy = R sin(x ) dA

=
=
=

x2

0
1
0
1
0

sin(x3 ) dy dx
y=x2

sin(x3 ) y]y=0 dx
sin(x3 )x2 dx

1
1
cos(x3 )]0
3
2
=
3

b.

We integrate with respect to

rst to obtain

2 x
2 x
R y e dA = 0 0 y e dy dx
1
2
= x3 ex dx
3 0
1
= tet dt
6 0
1
=
6
2

where we used the integration by parts

tet dt =

t d(et )

= lim ([tet ]c0 +


c

et dt)

c
= lim c lim [et ]c0
c e
c
1

= lim c lim (ec 1)


c e
c
=1
L'H

in the last step.

c.

This time we will use the polar coordinates. To do so we rst determine the

region of integration

R.

x= 0 to x = 2y y 2 as shown
2y y 2 x2 = 2y y 2
by the red line segment in the gure. Since x =

x2 + (y 1)2 = 12 , x = 2y y 2 gives the right semicircle of the circle x2 + (y 1)2 = 1.


On the other hand, the y -integral goes from y = 0 to y = 2. Therefore R is right half
of the disk x2 +(y 1)2 1. Note that the polar equation of the circle x2 +(y 1)2 = 1
is r = 2 sin , and to obtain the right semicircle we vary from = 0 to = /2.
In the iterated integral the

x-integral

goes from

75

Hence:

2yy 2

0 0

xy
xy
dx dy = 2
dA
2
+y
R x + y2
/2
2 sin r cos r sin
=
r dr d

r2
0
0

x2

=
=
=

/2

2 sin

0
/2
0
/2

sin cos r dr d

sin cos [

r2 r=2 sin
]
dr d
2 r=0

2 sin3 cos d

0
/2

sin4
=
]
2 0
1
=
2

d.

Again we use the polar coordinates. This time the integration region is

{(x, y) x 0 and y 0},


0

that is, the rst quadrant.

dy dx
1
=
dA

(x2 + y 2 )2 + 1
R (x2 + y 2 )2 + 1
/2
r dr d
=

r4 + 1
0
0

76

R=

/2

1
r=c
lim [arctan(r2 )]r=0 d
c
2
0
/2
=
d
4
0
2
=
8
=

21.

Evaluate the double integral

Solution:
the line

Let

y=x

1
R (x2 + y 2 )2 dA

where

R is the region shown in the gure.

be the portion of the region lying in the rst quadrant between

and the

x-axis.

By symmetry we have:

1
1
R (x2 + y 2 )2 dA = 8 R (x2 + y 2 )2 dA
/4
2 sec
1
= 8
r dr d

(r2 )2
0
sec
/4
1 r=2 sec
= 8
[ 2 ]
d
2r r=sec
0
= 3

/4

cos2 d

1 + cos 2
d
2
0
3
sin 2 /4
= [ +
]
2
2 0
3 3
=
+
8 4
= 3

/4

77

22.

r2 = 2 cos(2)
in the plane is the base of a solid right

2 r2 . Find the cylinder's volume.


sphere z =

The region enclosed by the lemniscate

cylinder whose top is bounded by the

Solution:

Let

R be the the region enclosed by the lemniscate in the rst quadrant.

By symmetry the volume is

2 r2 dA

= 4

2 cos 2

/4
0

2 r2 r dr d

r= 2 cos 2
1
4 /4 3/2
[ (2 r2 )3/2 ]
(2 (2 2 cos 2)3/2 ) d
= 4
d =
3
3 0
0
r=0

/4
8 2
2 2 32 /4 3
(1 (2 sin2 )3/2 ) d =
=

sin d

3
3 3 0
0

2 2 32 /4
2 2 32 1
2
=

(1 cos2 ) sin d =

(1 u ) du

3
3 0
3
3 1/ 2

1
3
2 2 32
u
2 2 32 2
5
=

[u ] =

( )
3
3
3 1/ 2
3
3 3 6 2

2 2 64 40 2
=

+
.
3
9
9
/4

78

23.

Let

D be the region in space bounded by


y = x2 on the sides, and the xy -plane

the plane

y+z = 1

on the top, the parabolic

V of the
region D in terms of iterated integrals with orders of integration (a) dz dy dx and (b) dx dy dz .
cylinder

at the bottom. Express the volume

Solution:

79

1
D (x2 + y 2 + z 2 )2 dV where D is the region bounded by the

cylinder x2 + y 2 = 1 on the sides and by the hemisphere z =


4 x2 y 2 at the bottom. Express

24.

Consider the triple integral

this integral in terms of iterated integrals in

a.

Cartesian coordinates,

b.

cylindrical coordinates,

c.

spherical coordinates, and

d.

evaluate the integral in the coordinate system of your choice.

Solution:
x2 + y 2

We observe that the projection of

1.

to the

xy -plane

is the unit disk

A vertical line passing through a point of the unit disk enters the region

at a point on the hemisphere


and remains in the region from there on. The equations

of the hemisphere z =
4 x2 y 2 and the cylinder x2 + y 2 = 1 are z = 4 r2 and

r = 1,

respectively, in cylindrical coordinates. The answers to parts

(a),

1x2

1
1
1
dV
=

D (x2 + y 2 + z 2 )2
1 1x2 4x2 y2 (x2 + y 2 + z 2 )2 dz dy dx ,
and

(b),
2
1

1
1
D (x2 + y 2 + z 2 )2 dV = 0 0 4r2 (r2 + z 2 )2 r dz dr d ,

immediately follow from these observations.


To do part

(c) we further observe that

the equations of the hemisphere and the cylinder are

=2

and

sin = 1 ,

respectively,
a ray starting at the origin enters the region at a point on the hemisphere and
leaves the region at a point on the cylinder, and
such a ray intersects the region exactly when
through a point on the intersection circle when

0 /6.
= /6.)

(The ray passes

Therefore,

2
/6
csc 1
1
2 sin d d d .
D (x2 + y 2 + z 2 )2 dV = 0 0 2
4

80

We use this last iterated integral for the computation

(d):

2
/6
csc 1
1
sin d d d
D (x2 + y 2 + z 2 )2 dV = 0 0 2
2
2
/6
1 =csc
=
[
]
sin d d
0
=2
0
2
/6
1
=
( sin + ) sin d d

2
0
0
2
/6
1 cos 2 sin
=
+
) d d
0 (
2
2
0
2
sin 2 cos =/6
=
[ +

]
d
2
4
2 =0
0

2
3
3 1

+ ) d
=
( +
12
8
4
2
0

3
= (1
)
6
4

81

25.

be the region in space bounded on the top by the sphere x2 + y 2 + z 2 = 2 and on the
bottom by the paraboloid z = x2 + y 2 . Express the volume V of D in terms of iterated integrals
Let

in the

(a) Cartesian, (b) cylindrical, and (c) spherical coordinates.

Solution:

Note that the projection of the curve of intersection of the sphere and
x2 + y 2 = 1 in the xy -plane. This curve also bounds

the paraboloid is the unit circle


the projection of the solid

V =

to the

Hence we have

2x2 y 2

1x2

1
1

xy -plane.

1x2 x2 +y2

and

V =
0

dz dy dx

2r2

0 r 2

r dz dr d .

D through the sphere if the


angle it makes with the positive z -axis is less than /4, whereas it does so through
the paraboloid if this angle is between /4 and /2. Hence:

Now observe that a ray starting at the origin leaves

V =
0

/4

2 sin d d d

2
0

/2

/4 0

82

cos / sin2

2 sin d d d

26.

Consider the iterated integral

/2

2r2

0 r

dz dr d

in cylindrical coordinates.

a.

Change the order of integration into

b.

Express the integral in spherical coordinates with order of integration

Solution: a.
z = 2

x2

y2.

z = r

is the cone

z 2 = x2 + y 2 ,

and

z = 2 r2

d d d .

is the paraboloid

These surfaces intersect along a circle that is also the curve of

intersection of the cylinder

0 /2,

dr dz d.

r=1

and the horizontal plane

the integration region

z = 1.

Since

0r1

and

is the region in the rst octant bounded by

the paraboloid on the top and the cone at the bottom.

When we integrate with respect to

rst, we will be moving along a curve on

83

which

and

are constant. These two conditions describe a horizontal plane and a

z -axis, respectively. Therefore the r-integration is along a ray


perpendicular to the z -axis like the blue ones in the gure. Such a ray starts on the
z -axis in D, and leaves D through the cone if 0 z 1, and through the paraboloid
if 1 z 2. Therefore,
plane containing the

/2

2r2

0 r

dz dr d =

/2

0 0 dr dz d

+
where we used the fact that

b.

We have

/2

0
as

z = 2 r2

dV = r dz dr d

(a).

and

2r2

0 r

and

r=

in cylindrical coordinates where

2z

1 0

dz dr d =

Since we want to integrate with respect to


on which

r0

/2

dr dz d

2z.

1
dV
r

is the region described in part

rst, we will be moving along the curves

are constant. These two conditions describe a sphere with center

z -axis, respectively. Therefore the


-integration takes place along a vertical semicircle subtended by a diameter along
the z -axis and with center at the origin like the green ones in the gure. Such a
semicircle starts on the positive z -axis in D , and leave the region of integration

intersecting the cone for 0


2 and the paraboloid for 2 2. The upper
half of the cone z = r has the equation = /4 in spherical coordinates. On the

2
other hand, z = 2 r
cos = 2 ( sin )2 cos = (1 + 42 7)/(2) for
a point on the paraboloid as /4 . Therefore,

at the origin and a half-plane whose spine is the

1
1
D r dV = D sin dV
=

/2
0

/4

0
+

where we substituted

d d d
/2

2 0

dV = 2 sin d d d

coordinates.

84

arccos((1+ 42 7)/(2))

d d d
for the volume element in spherical

27.

The region

about the

z -axis

to obtain a solid

iterated integrals in

Solution:

z 2 = y 2 y 4 in the right half of the yz -plane is rotated


the xyz -space. Express the volume V of D in terms of

bounded by the curve

in

(a) the Cartesian, (b) the cylindrical and (c) the spherical coordinates.

Since the surface bounding the the solid

is obtained by revolving a

z -axis, its equation in the cylindrical coordinates will not depend


on . Since r = y in the yz -plane and the curve has the equation z 2 = y 2 y 4 , we
conclude that z 2 = r 2 r 4 is the equation of the surface in the cylindrical coordinates.
curve about the

From this

V =
0
follows as the projection of

r2 r4

0 r2 r4 r dz dr d

to the

xy -plane

is the unit disk.

The equation z 2 = r 2 r 4 in the cylindrical coordinates transforms to z 2 = (x2 +


y 2 ) (x2 + y 2 )2 in the Cartesian coordinates, and once again using the fact that the
projection of

to the

xy -plane

is the unit disk, one now obtains:

1x2

(x2 +y 2 )(x2 +y 2 )2

V =
(x2 +y2 )(x2 +y2 )2 dz dy dx
1 1x2
The spherical coordinates require a little bit more work. First note that the equation
z 2 = r2
r4 in the cylindrical coordinates now gives ( cos )2 = ( sin )2 ( sin )4 ,
or =
sin2 cos2 / sin2 , in the spherical coordinates. Next note that for
2
sin cos2 = cos 2 to be nonnegative, must be between /4 and 3/4 in the
interval

[0, ].

Therefore:

V =
0

Remark:

sin2 cos2 / sin2

3/4

/4

Compare this example with

2 sin d d d

Example 45 in Part 1.

85

28.

0
Evaluate the iterated integral

2y+3

1 y

x+y
ex2y dx dy .
2
(x 2y)

Solution:

29.

Let

(u, v)

and

Solution:

(x, y)

be two coordinate systems. Show that

By denition,

(x, y) xu xv
= xu yv xv yu
=
(u, v) yu yv
and

(u, v) ux uy
= ux vy uy vx .
=
(x, y) vx vy
86

(x, y) (u, v)

= 1.
(u, v) (x, y)

Therefore,

(x, y) (u, v)

= (xu yv xv yu )(ux vy uy vx )
(u, v) (x, y)
= xu ux yv vy + xv vx yu uy xu uy yv vx xv ux yu vy
= xu ux yv vy + xv vx yu uy xu uy yv vx xv ux yu vy
+ xv vx yv vy + xu ux yu uy xv vx yv vy xu ux yu uy
= (xu ux + xv vx ) (yu uy + yv vy )
(xu uy + xv vy ) (yu ux + yv vx )
= xx yy xy yx
=1100
=1

30.

Evaluate the double integral

Solution:

R e

x2 /y

dA

where

R = {(x, y) x2 y

x }.

u = y 2 /x, v = x2 /y . In this coordinate


G = {(u, v) 0 u 1 and 0 v 1} .

We change the coordinates to

system the region of integration becomes

We have

(u, v) ux uy
y 2 /x2 2y/x
=
=
= 3
2x/y x2 /y 2
(x, y) vx vy

1
(x, y)
1
= ,
=
(u, v) (u, v)
3
(x, y)

and the change of variables formula gives

x2 /y

R e

dx dy = ev
G

1
1
1
e1
(x, y)
du dv = ev dv du =
.
(u, v)
3
3
0
0

87

Remark:
any

We can multiply dierentials. The rules are

uv -coordinate

du du = 0 = dv dv

and

dv du = du dv

in

system. For instance, when changing from Cartesian to polar coordinates,

we have

dx dy = d(r cos ) d(r sin )


= (cos dr r sin d)(sin dr + r cos d)
= r sin2 d dr + r cos2 dr d
= r(sin2 + cos2 ) dr d
= r dr d
and in

Example 30 we have
du dv = d(y 2 /x) d(x2 /y)
= (2y/x dy y 2 /x2 dx)(2x/y dx x2 /y 2 dy)
= 4 dy dx + dx dy
= 4 dx dy + dx dy
= 3 dx dy ,

hence

1
dx dy = du dv .
3

This can be used to keep track of how the area element changes under

a coordinate change, but note that the sign of the factor in front must be corrected by hand so
that it is positive on the region of integration.

31.
C

Consider the transformation

T x=

such that the inequality

u
v
,y=
u+v+1
u+v+1

. Show that there is a constant

(x, y)
G (u, v) du dv C
holds for all regions

contained in the rst quadrant of the

uv -plane.

u
v
u+v
0, y =
0 and x + y =
1
u+v+1
u+v+1
u+v+1
x
y
for u 0 and v 0. We also have u =
and v =
. Therefore T maps
1xy
1xy
the rst quadrant of the uv -plane into the triangle R = {(x, y) x+y 1, x 0, y 0}

Solution:

Observe that

x=

in a one-to-one manner.

88

Hence:

(x, y)
G (u, v) du dv = T (G) dx dy = (Area

Remark:

of

T (G)) (Area

of

R) =

1
2

One can also do a straightforward computation.

1
(x, y) xu xv
(v + 1)/(u + v + 1)2
u/(u + v + 1)2
=
=
=
2
2
v/(u + v + 1)
(u + 1)/(u + v + 1)
(u, v) yu yv
(u + v + 1)3
and hence

du dv
(x, y)
1 dv
1

du
dv

=
G (u, v)
0 0 (u + v + 1)3 2 0 (v + 1)2 = 2 .

32.

Compute the Jacobian

(x, y, z)
(, , )

where

(, , ) are the spherical coordinates and (x, y, z)

are the Cartesian coordinates.

Solution:

We have

x = sin cos , y = sin sin , z = cos .

Therefore

x x x
(x, y, z)
= y y y
(, , )
z z z
sin cos cos cos sin sin
= sin sin cos sin sin cos
cos
sin
0
= ( sin sin )

sin sin cos sin

cos
sin

( sin cos )

sin cos cos cos

cos
sin
+0

sin cos cos cos

sin sin cos sin

= ( sin sin )( sin )(sin2 + cos2 )


( sin cos )( cos )(sin2 + cos2 )
+0
2
2
2
= sin (sin + cos )
= 2 sin
where we used the cofactor expansion with respect to the third column.

89

Part 3: Sequences and Series

1. Let a1 = 1, a2 = a3 = 2, a4 = a5 = a6 = 3, a7 = a8 = a9 = a10 = 4,
an 1, 2, 2, 3, 3, 3, 4, 4, 4, 4, 5, 5, 5, 5, 5, 6, . . . . What is a2015 ?

and so on.

That is,

Solution:

an = k if 1 + 2 + + (k 1) < n 1 + 2 + + k . In other words, an = k if


k(k 1)/2 < n k(k + 1)/2. Since for k = 63, k(k 1)/2 = 1953 and k(k + 1)/2 = 2016,
we have a2015 = 63.

Remark:
2.

A more explicit formula

8n + 1 1
an =

can be obtained.

Curve 1 is an equilateral triangle with unit sides. For

n 2, Curve n is obtained from Curve n1

by replacing the middle third of every edge with the other two sides of the outward pointing

Ln be the
Ln , An , lim Ln

equilateral triangle sitting on it. Let

length of Curve n and

region enclosed by Curve n . Find

and

Solution:

Let

en

and

dn

An

be the area of the

lim An .

denote the number of edges and the length of each edge

en = 4en1 and dn = dn1 /3 for n 2, and e1 = 3


and d1 = 1, we nd that en = 3 4n1 and dn = 1/3n1 for n 1. It follows that
Ln = en dn = 3 (4/3)n1 for n 1, and lim Ln = lim 3 (4/3)n1 = .
of Curve n , respectively.

Since

On the other hand, the

region is obtained by adjoining

en1

equilateral triangles

(n 1) region. Therefore,

3 (dn1 /3)2
3 1 2
An = An1 + en1
= An1 + 3 4n2
(
)
4
4 3n1

of side length

dn1 /3

nth

and then

to the

st

1
4 n2
4 n3
An = (( ) + ( ) + + 1) + A1
9
4 3 9

3
A1 =
, this gives
4

3 3
4 n1
3
An =
(1 ( ) ) +
20
9
4

for

n 2.

Since

for

n 1.

We obtain

3 3
3 2 3
lim An =
+
=
.
n
20
4
5

90

Remark:

Curve 1 :

Curve 2 :

Curve 3 :

Curve 4 :

t = 0 sec, takes out 1 ball at t = 1/2 sec,


takes out 1 ball at t = 7/8 sec, puts 3 balls at t = 15/16 sec, takes out

A magician puts 1 ball into an empty box at

t = 3/4 sec,
t = 31/32 sec, and
box at t = 1 sec.

puts 2 balls at
1 ball at
in the

so on. Then she challenges you to guess how many balls there are

You ask for the advice of your friends.

Friend1 says: The net result of

2k + 1st

and

2k + 2nd

steps for

k1

is to put at

least one more ball into the box. There are innitely many balls in the box at

t=1

sec."

ball1, ball2, ball3, and so on


ball1 at t = 0
sec, takes out ball1 at t = 1/2 sec, puts ball2 and ball3 at t = 3/4 sec, takes out
ball2 at t = 7/8 sec, puts ball4, ball5, and ball6 at t = 15/16 sec, takes out ball3
at t = 31/32 sec, and so on. For any given k , ballk is not in the box at time t = 1
Friend2 says: Imagine that the balls are labeled as

with invisible ink which only the magician can see. Then she puts

sec, because it was taken out at time


box at time

t=1

t = 1 1/22k1

sec."

What do you think?

91

sec. There are no balls in the

3.

Let the sequence

{an }

be dened by

a1 = 1

and

an =

sequence converges and nd its limit.

Solution:

for

n 1.

Show that the

0 < 1 + an < 1 + an+1 and an+1 =


1/(1 + an ) > 1/(1 + an+1 ) = an+2 . Similarly, if an > an+1 > 0, then 1 + an > 1 + an+1 > 0
and an+1 = 1/(1 + an ) < 1/(1 + an+1 ) = an+2 . Since a1 = 1 > 1/2 = a2 , it follows that
Observe that if

0 < an < an+1 ,

1
1 + an1

then

1 = a1 > a3 > a5 > > a6 > a4 > a2 =

1
.
2

Therefore, the sequence {a2n }n=1 is increasing and bounded from above by 1, and

the sequence {a2n1 }n=1 is decreasing and bounded from below by 1/2. By the
Monotonic Sequence Theorem, then both of these sequences are convergent.

lim a2n1 = L and lim a2n = M . Taking the limit of a2n+1 = 1/(1 + a2n ) as n
n
n
we obtain L = 1/(1 + M ), and taking the limit of a2n = 1/(1 + a2n1 ) as n we
Let

From M + M L = 1 and LM +

Therefore the sequence {an }n=1 converges to L = M .


obtain

M = 1/(1 + L).

L=1

it follows that

M = L.

L2 + L 1 = 0 gives L = ( 5 1)/2
or L = ( 5 + 1)/2. Since 0 < an
n 1, we must have 0 L . Hence L = ( 5 1)/2 is the limit and

51
lim an =
.
n
2

Finally,

Remark:

This result is sometimes expressed symbolically in the form:

51
=
2

1
1

1+

1+

1+

1+
1+

Remark:

1
1+

xn = Fn /Fn+1 for n 1 where Fn 1, 1, 2, 3, 5, 8, . . .


showed that lim Fn /Fn+1 = ( 5 1)/2 .

Note that

sequence. So we

Remark:

for all

is the Fibonacci

It can be showed with a little bit more work that any sequence satisfying the given

(5 1)/2 if a1 =/ (1 + 5)/2 and a1 =/ Fn+1 /Fn for any n 1 .


5)/2 , then the sequence is constant; and if a1 = Fn+1 /Fn for
On the other hand, if a1 = (1 +
some n 1, then an = 1 and an+1 is undened.
recursion relation converges to

92

4.

Let the sequence

{xn }

be dened by

x0 = 2

and

xn =

xn1
1
+
2
xn1

a.

Find the limit of this sequence assuming it exists.

b.

Show that the limit exists.

Solution: a.
xn =

We assume that the limit

xn1
1
+
2
xn1

n1

for

L = lim xn

xn xn1 =

for

n 1.

exists. Then

x2n1
+1
2

for

n1

x2n1
L2
+ 1 L2 =
+1
2
2

L = 2 or L = 2.

lim(xn xn1 ) = lim


as

lim xn1 = lim xn = L .

Therefore

L2 = 2

x0 = 2 > 0 and if xn > 0 then xn+1 = xn /2 + 1/xn > 0 + 0 = 0. Hence by induction


xn > 0
for all n 0. It follows that L = lim xn 0. We conclude that the limit is
2.

b.

In part

(a) we proved that xn > 0 for all n 0.

Therefore

{xn } is bounded from

below.
Now we will show by induction on

Let

n = 0.

Let

n>0

that

2 < xn+1 < xn

x0 = 2, x1 = 3/2, and x21 = 9/4 > 2,

assume that
2 < xn+1 < xn . Then

Since
and

xn+1 xn+2 =
and hence

xn+2 < xn+1 .

and since

xn+2 > 0,

we have

n 0.

2 < x1 < x0 .

x2 2
xn+1
1

= n+1
>0
2
xn+1
2xn+1

On the other hand,

x2n+2

for all

x2 2
xn+1
1
2=(
+
) 2 = ( n+1
) >0
2
xn+1
2xn+1

we have

xn+2 >

2.

Therefore the sequence is decreasing.


Since the sequence is bounded from below and decreasing, we conclude that it
converges by the Monotonic Sequence Theorem.

93

5.

For each of the series in

the

nth

partial sum

a.

sn

(a-d), determine whether there exists a positive integer n such that

of the series satises the condition

1
n
n=1 2

b.

c.

n=1

Solution:

2014 sn 2015.
n

2999
)
(
n=1 3000

d.

1
n=1 n

Note that all terms of these series are positive, and therefore, their partial

sums form increaisng sequences. This will be used repeatedly in the following.

a.

We have

1
1
1/2
= 1 < 2014
<
=

k
n
1 1/2
n=1 2
k=1 2
n

sn =
for all

n 1,

where we used the geometric series sum formula, and hence there are

no partial sums lying between 2014 and 2015.

b.

This time we have

4
1
1

= 5 + 25 + 125 + 625 = 780 < 2014

k
k
k=1 2
k=1 2

sn =
for

n 4,

and

5
1
1

= 5 + 25 + 125 + 625 + 3125 = 3905 > 2015

k
k
k=1 2
k=1 2

sn =
for

n 5.

c.

Every term of this series is between 0 and 1, hence its partial sums increase in

Therefore no

sn

lies between 2014 and 2015.

s1 = 2999/3000 < 2014.

steps smaller than 1 staring with

Moreover,

2999
2999/3000
) =
= 2999 > 2015
(
1 2999/3000
n=1 3000
by the geometric series sum formula.
satisfying the condition

d.

Hence there is at least one partial sum

2014 sn 2015.

The same reasoning as in part

sn

(c) works, this time with the observation that

1
= > 2015 ,
n=1 n

to give the existence of at least one partial sum

sn 2015.

94

sn

satisfying the condition

2014

6.

Determine whether each of the following series converges or diverges.

a.
d.
g.

(21/n 21/(n+1) )
n=1

nn
n=1 n!

1
n=3 n ln n ln(ln n)

Hence

b.

We have

lim sn = 1 .

n ln n
n
n=2 3

e.

n sin n
n=1 n +

sin n
2
n=1 n

c.

f.

h.

Solution: a.

b.

k=1

k=1

(1)n+1 cos ( )
n
n=1
n
n
5 2
n
n
n=1 7 6

sn = (21/k 21/(k+1) ) = 21/k 21/(k+1) = 2 21/(n+1) .

Therefore the series converges and

k=1

(21/n 21/(n+1) ) = 1 .
n=1

We have

an = n ln n/3n

and

an+1
(n + 1) ln(n + 1)/3n+1 1
n+1
ln(n + 1)
= lim
=

lim

lim
n an
n
n ln n/3n
3 n n n ln n

= lim

1
ln(x + 1) 1
1/(x + 1) 1
1
= lim
1 lim
= 1= .
x
3
ln x
3 x 1/x
3
3
L'H

=
Since

c.

= 1/3 < 1 ,

the series

n ln n
n
n=2 3

an = (1)n+1 cos(/n),
lim an = lim cos(/n) = cos 0 = 1 .
n
n
by the nth Term Test.

d.

Since

We have

an = nn /n!

converges by the Ratio Test.

we

have

Therefore

an = cos(/n)
lim an =/ 0 , and

for

n 2,

and

the series diverges

and

an+1
(n + 1)n+1 /(n + 1)!
1
= lim
= lim
= lim (1 + ) = e .
n
n an
n
n
n /n!
n
Since

e.

the series

nn
n=1 n!

diverges by the Ratio Test.

We observe that

Since

1/(n + n sin n)
L = lim
= lim
n
n
1/n

0<L<

n sin n
n=1 n +

= e > 1,

1
1
=
=1.
sin n 1 + 0
1+
n

and the harmonic series

1
n=1 n

diverges, we conclude that the series

diverges by the Limit Comparison Test.

95

f.

(5/7)n

The geometric series

have

L = lim

series

n
n
5 2n

n=1

g.

Since

on

n=1
(5n 2n )/(7n 6n )

7n 6n
ln x

(1, ).

5n /7n

converges as

r = 5/7

r = 5/7 < 1.

1 (2/5)n 1 0
=
= 1.
n 1 (6/7)n
10

= lim

Since

We also

0 < L < ,

the

converges by the Limit Comparison Test.

(0, ) , ln(ln x) is an increasing function


positive on [3, ) . Therefore x ln x ln(ln x) is

is an increasing function on

Both

ln x

ln(ln x) are
[3, ) as it is the product of three positive and increasing
implies that 1/(x ln x ln(ln x)) is a positive and decreasing

and

positive and increasing on


functions. This in turn
function on

[3, ) .

Since it is also continuous, we can apply the Integral Test. The

improper integral

dx
du
=
x ln x ln(ln x)
ln(ln 3) u

diverges, where we used the change of variable

the series

h.

n=3 n ln n ln(ln n)

Hence

diverges.

Consider the series

u = ln(lnx), du = dx/(x ln x) .

1 + sin n
.
n2
n=1

Since

0 1 + sin n 2

for

n 1,

we have

1 + sin n
2
1

for n 1 . The p-series


converges as p = 2 > 1 . Therefore
2
2
2
n
n
n=1 n

2
1
1 + sin n
the series
=
2
converges. Then the series
converges by the

2
2
n2
n=1 n
n=1 n
n=1

1 + sin n
1
Direct Comparison Test. Since both
and
converge, the series
2
2
n
n=1
n=1 n

sin n
2 , which is their dierence, also converges.
n=1 n

Remark:

Other tests can be used too. Here are some examples:

1
2
n=1 n

In part

(a), the Limit Comparison Test with the series

In part

(b), the Limit Comparison Test with the geometric series

In part

(d), the Root Test also works where we use the fact that n
lim

96

also works.

1
n
n=1 2

also works.

1/n

(n!)
n

1
e

Remark:

The following was a bonus problem on Moodle in Spring 2010 Math 102 course.

Problem K: In class we showed that 0.12 = 0.121212. . .

= 12/99 as an application

of the geometric series.

x = 0.121212 . . . , then 100x =


therefore x = 12/99 . Of course

We learn the following trick in elementary school: If

12.121212 . . . ;

their dierence gives

99x = 12

and

when we see this in elementary school, no one talks about convergence.


That is what we are going to do in this problem. We will forget about convergence
as we know it, and take a trip to Tersonia.
In Tersonia after they teach the students about integers and rational numbers,
they come to the decimal representations of numbers. As you know our decimal
expansions have the form

dn dn1 . . . d2 d1 d0 .d1 d2 . . .
where each

di

is in

{0, 1, . . . , 9}.

We can have innitely many nonzero digits after

the decimal point, but we must have only nitely many nonzero digits before the
decimal point. In Tersonia they do just the opposite. Their decimal expansions
have the form

. . . t3 t2 t1 t0 .t1 t2 . . . tn
where each

ti

is in

{0, 1, . . . , 9}.

Note that there is no minus sign. They can have

innitely many nonzero digits before the decimal point, but they can only have
nitely many digits after the decimal point.
Take a few minutes to convince yourself that Tersonians can add and multiply
their decimal expansions just like we do.
Why no minus sign? Well, because Tersonians don't need it. Negative numbers

y = 12.0 = . . . 121212.0 . Then


y = 12/99 . So in fact 12.0 is a

are already there. For instance, consider the number

100y = . . . 121200.0

and

99y = 12 .

Therefore

negative number.
Here are some problems from Tersonian Elementary School Mathematics Book :

Part

(e)

a.

1
=?
2

e.

Find two nonzero numbers

b.

1
=?
3
A

and

c.

1
=?
7

such that

was later turned into a programming challenge.

d.
A B = 0.

A Java applet that computes

last n digits of A and B when their last digits are


http://www.fen.bilkent.edu.tr/otekman/calc2/ters.html .

the

97

1 = ?

given

can

be

found

at

7.

Determine whether the sum of the series

Solution:

Let

bn =

4n
n!(n + 1)!

for

(4)n
n=0 n!(n + 1)!

n 0.

is positive or negative.

Then:

b >0 n0
lim b = lim n!(n4+ 1)! = lim 4n! lim (n +1 1)! = 0
for all

as both limits are 0, the rst

one being one of the  Useful Limits .

n bn+1

4n
4n+1

(n + 2)(n + 1) 4 n 1.
n!(n + 1)! (n + 1)!(n + 2)!

Therefore the series satises the conditions of the Alternating Series Test.

In

particular, it converges. Moreover,

(4)n
4
42
43
44
4 4 4
=1
+

+
=12+ +

1!2! 2!3! 3!4! 4!5!


3 9 45
n=0 n!(n + 1)!

S=

s3 =1/9
and, by the Alternating Series Estimate,

1/45,

Remark:

and

In other words,

S (1/9) <

is negative.

(4)n
n=0 n!(n + 1)!

S s3 < b4 .

b4

is

1
J1 (4),
2

(1)n (x/2)2n+1
is the
n!(n + 1)!
n=0
approximately 0.03302166401.

where

the rst kind of order 1, and its value is

J1 (x) =

98

Bessel function of

8.

Find the radius of convergence of the power series

(1)n
n=0

Solution:

xn
.
(2n + 1)(n2 + 1)

We will give two dierent solutions.

Solution 1 : Here we will use the Ratio Test.


We have

an = (1)n

xn
(2n + 1)(n2 + 1)
= lim

and

an+1
an
(1)n+1

= lim

xn+1

(2n+1 + 1)((n + 1)2 + 1)


xn

(2n + 1)(n2 + 1)

(1)n

xn+1
(2n+1 + 1)((n + 1)2 + 1)
= lim
n
xn
(2n + 1)(n2 + 1)
1 + 2n
n2 + 1
= lim (

)x
n 2 + 2n (n + 1)2 + 1
x
=
.
2
If

x < 2 ,

then

= x/2 < 1

Test. On the other hand, if

and the power series converges absolutely by the Ratio

x > 2 ,

then

= x/2 > 1

and the power series diverges

by the Ratio Test.


It follows by the denition of the radius of convergence that

R = 2.

Solution 3 : Here we will use the radius of convergence formulas.

We have

cn =

(1)n
,
(2n + 1)(n2 + 1)

and the radius of convergence formula gives

cn+1
1/((2n+1 + 1)((n + 1)2 + 1))
1
= lim
= lim
n
R n cn
1/((2n + 1)(n2 + 1))
n2 + 1
1 + 2n
1

= lim (
)
=
.
n 2 + 2n (n + 1)2 + 1
2
Therefore

R = 2.

99

9.

(1)n

Consider the power series

n=0

a.

x2n+1
(2n + 1)(n2 + 1)

Find the radius of convergence of the power series.

b.

Determine whether the power series converges or diverges at the right endpoint of its

interval of convergence.

c.

Determine whether the power series converges or diverges at the left endpoint of its

interval of convergence.

Solution: a.

We have

an = (1)n

x2n+1
(2n + 1)(n2 + 1)

and

= lim an 1/n
n

1/n

x2n+1
= lim ( n
)
n (2 + 1)(n2 + 1)
x2 x1/n
= lim
n 2 (1 + 2n )1/n n1/n (1 + n2 )1/n
2

=
If

x <

2,

Root Test;
Root Test.

b.

At

x=

x
.
2

2
= x
/2 < 1 and the power series
and if x >
2
, then = x2 /2 > 1 and the
Therefore R =
2.
then

converges absolutely by the


power series diverges by the

we have

2n+1
x
(
2)2n+1
n
=
(1)
(1)n n

(2 + 1)(n2 + 1) n=0
(2n + 1)(n2 + 1)
n=0

2n
= 2 (1)n n
.
(2 + 1)(n2 + 1)
n=0

Consider

the

corresponding

absolute

value

series

2n
.
Since
n
2
n=0 (2 + 1)(n + 1)

1
p-series 2 with p = 2 > 1
n=1 n

2n
1
for all n 1 and the
<
(2n + 1)(n2 + 1) n2

2n
converges,
converges by the Direct Comparison Test; and then
n
2
n=0 (2 + 1)(n
+ 1)
the power series at x =
2 converges absolutely by the Absolute Convergence Test.
0<

c.

At

x= 2

we have

x2n+1
( 2)2n+1
n
= (1) n
(1) n
(2 + 1)(n2 + 1) n=0
(2 + 1)(n2 + 1)
n=0

2n
= 2 (1)n+1 n
.
(2 + 1)(n2 + 1)
n=0

100

This is just

1 times the series we considered in part (b), and therefore it converges

absolutely.

10.

Determine the radius of convergence and the interval of convergence of the power series

xn
.
n+1
n=0 3n + (1)

Also determine the type of convergence at each point of the interval of convergence.

Solution:

As

cn =

1
3n + (1)n+1

, the formulas for the radius of convergence gives

cn+1
1/(3(n + 1) + (1)n+2 )
3 + (1)n+1 /n
1
= lim
= lim
=
lim
= 1.
R
cn
1/(3n + (1)n+1 )
3 + 3/n + (1)n+2 /n
Therefore

x > 1.
At

x=1

R = 1.

Hence we have absolute convergence for

we have

x < 1

and divergence for

xn
1
=
.

n+1
n+1
n=0 3n + (1)
n=0 3n + (1)

Since

1/(3n + (1)n+1 )
1
1
= lim
=
n
n 3 + (1)n+1 /n
1/n
3

L = lim

is a positive real number and the harmonic series


diverges by the Limit Comparison Test at
At

x = 1

we have

x = 1.

1
n=1 n

diverges, the power series

xn
(1)n
=
.

n+1
n+1
n=0 3n + (1)
n=0 3n + (1)

Let

i.
ii.

iii.

un =

1
3n + (1)n+1

1
> 0 for all n 1 .
3n + (1)n+1
0 < 3n + (1)n+1 3n + 1 < 3n + 2 3(n + 1) + (1)n+2
un > un+1 for n 1 .
1
= 0.
lim un = lim
n
n 3n + (1)n+1
un =

for all

n 1.

Hence

It follows that the series

(1)n
n+1
n=0 3n + (1)

converges by the Alternating Series Test.

Its absolute value series is the same as the series

1
n+1
n=1 3n + (1)

1+

and we showed

that this series diverges. Hence the power series converges conditionally at

101

x = 1 .

R = 1,

To summarize, the radius of convergence is


is

[1, 1),

converges conditionally at

11.

the interval of convergence

the power series converges absolutely at every point of

x = 1 .

(1, 1),

and it

Consider the power series

a.

1 3 5 (2n 1) n
x .
n=1 2 4 6 (2n)

f (x) = 1 +

Show that the radius of convergence of the power series is

b.

R = 1.

Determine the behavior of this power series at the endpoints of its interval of

convergence.

c.

Show that

d.

Solve this dierential equation to show that

e.

Show that

2(1 x)f (x) = f (x)

for

x < 1.
f (x) =

1
1x

for

x < 1.

1 3 5 (2n 1) x2n+1

2n + 1
n=1 2 4 6 (2n)

arcsin x = x +
for

x < 1.

Solution: a.

We have

cn =

1 3 5 (2n 1)
2 4 6 (2n)

for

n 1.

We use the radius of

convergence formula

1 3 5 (2n 1)(2n + 1)
1
cn+1
2n + 1
2 4 6 (2n)(2n + 2)
= lim
= lim
= lim
=1
n
n 2n + 2
1 3 5 (2n 1)
R n cn
2 4 6 (2n)
to obtain

b.

At

R = 1.

x=1

we have the series

1 3 5 (2n 1)
n=1 2 4 6 (2n)

1+

. Since

1 3 5 (2n 1) 3 5
2n 1 1
1
=

>
2 4 6 (2n)
2 4
2n 2 2n 2n

for

1
n=1 n

n > 1 and the harmonic series

diverges, we conclude that the series at

x=1

diverges by the Direct Comparison Test.

At

x = 1

un =

we obtain the alternating series

1 3 5 (2n 1)
.
2 4 6 (2n)

1 + (1)n
n=1

We have

(i ) un > 0

102

for all

1 3 5 (2n 1)
2 4 6 (2n)

n 0,

and we also have

with

(ii )

un > un+1

for all

n0

as

un /un+1 = (2n + 2)/(2n + 1) > 1

for

n 0.

On the other

hand,

12 32 52 (2n 1)2 1 3 3 5
(2n 3)(2n 1) 2n 1 1
1
= 2 2

<
2
2
2
2
2
2 4 6 (2n)
2
4
(2n 2)
2n
2n 2n

for n > 1. Therefore 0 < un < 1/ 2n for n > 1, and the Sandwich Theorem gives
(iii ) lim un = 0 . We conclude that the series at x = 1 converges by the Alternating
u2n =

n
Series Test. The convergence is conditional as we have already seen that the absolute
value series diverges.

c.

For

x < 1

we have

1 3 5 (2n 1) n
x .
n=1 2 4 6 (2n)

f (x) = 1 +
Dierentiating this we get

f (x) =
for

x < 1 .

1 1 3 5 (2n 1) n1
+
x
2 n=2 2 4 6 (2n 2) 2

Therefore

1 3 5 (2n 1) n
x
n=2 2 4 6 (2n 2)

2xf (x) = x +
and

2f (x) = 1 +
for

x < 1 .

3
1 3 5 (2n + 1) n
x+
x
2
n=2 2 4 6 (2n)

Taking the dierence of these two, we obtain

1
1 3 5 (2n 1) 2n + 1
x+
(
1) xn
2
2n
n=2 2 4 6 (2n 2)

1 3 5 (2n 1) 1 n
1
=1+ x+

x
2
n=2 2 4 6 (2n 2) 2n

1
1 3 5 (2n 1) n
x
=1+ x+
2
n=2 2 4 6 (2n)

1 3 5 (2n 1) n
=1+
x
n=1 2 4 6 (2n)
= f (x)

2(1 x)f (x) = 1 +

for

x < 1.

103

d.

For

x < 1,

2(1 x)f (x) = f (x)

Now substituting

e.

x=0

gives

We have

for

x < 1 .

Substituting

for

x2 < 1 .

f (x)
1
=
f (x) 2(1 x)
1
ln f (x) = ln 1 x + C for some constant C
2
A
f (x) =
for some constant A .
1x

1 = f (0) = A.

Therefore,

f (x) =

1
1x

for

x < 1.

1
1 3 5 (2n 1) n
=1+
x
1x
n=1 2 4 6 (2n)

x2

for

1
1 x2

gives

1 3 5 (2n 1) 2n
x
n=1 2 4 6 (2n)

=1+

Integrating this we obtain

1 3 5 (2n 1) x2n+1

+C
2n + 1
n=1 2 4 6 (2n)

arcsin x = x +
for

x < 1 .

Substituting

x=0

gives

0 = arcsin 0 = C ,

and hence

C = 0.

1 3 5 (2n 1) x2n+1

2n + 1
n=1 2 4 6 (2n)

arcsin x = x +
for

x < 1 .

Remark:

It can be shown that

for

1 x < 1 ,

and

1 3 5 (2n 1) n
1
=1+
x
1x
n=1 2 4 6 (2n)

1 3 5 (2n 1) x2n+1

2n + 1
n=1 2 4 6 (2n)

arcsin x = x +
for

x 1 .

104

Thus

12.

Consider the power series

n(2x 1)3n+1
5n
n=1

a.

Find the radius of convergence of the power series.

b.

Find the sum of the power series explicitly.

Solution: a.

n(2x 1)3n+1
5n

an =

gives

an+1
an
(n + 1)(2x 1)3(n+1)+1 /5n+1
= lim
n
n(2x 1)3n+1 /5n
n + 1 2x 13
= lim

n n
5
2x 13
=
.
5

= lim

< 1 and the series converges by the Ratio Test; and


> 1 and the series diverges by the Ratio Test. Since
3
1
5
2x 13 /5 < 1 x <
, it follows that the radius of convergence of the
2
2

3
5/2 .
power series is R =

If
if

2x 13 /5 < 1 ,
2x 13 /5 > 1 ,

b.

then

then

We know that

xn =
n=0
for

x < 1 .

Dierentiating this we obtain

nxn1 =
n=1
for

x < 1 .

1
1x

Now we replace

with

1
(1 x)2

(2x 1)3 /5

to get

n(2x 1)3(n1)
=
5n1
n=1

for

(2x 1)3 /5 < 1 .

for

x 1/2 <

(2x 1)3
(1
)
5

Finally we multiply by

n(2x 1)3n+1
=
5n
n=1

(2x 1)4 /5

(2x 1)4 /5
2

(1

(2x 1)3
)
5

5/2 .

105

to obtain

(2x 1)4
5

4 (4x3 6x2 + 3x 3)2

13.

Consider the function dened by:

xn

n=0

n! 2n(n1)/2

f (x) =

a.

Find the domain of

b.

Evaluate the limit

c.

Show that

d.

Show that

f.

f (x) ex
x0 1 cos x
3
f (2) < e + .
2
lim

f (2) < 0 .

Solution: a.

Since

cn =

1
n! 2n(n1)/2

, the radius of convergence formula gives

1
cn+1
1/((n + 1)! 2(n+1)n/2 )
1
= lim
= lim
= lim
=0.
n(n1)/2
n
n (n + 1)2n
R n cn
1/(n! 2
)
Therefore

b.

R = .

This means that the domain of

Using

xn

n=0

n! 2n(n1)/2

f (x) =

=1+x+

is

(, ) .

x2
x3
+ 3
+
2 2! 2 3!

we obtain

f (x) ex
= lim
x0 1 cos x
x0

lim

c.

(1 + x +

x2
x3
x2 x3
+ 3
+ ) (1 + x +
+
+ )
2 2! 2 3!
2! 3!

x2 x4
+
)
2! 4!
1
1 7
7 3
x2

x
x
4
48
4
48
= lim
= lim
1 4
1 2
x0 1
x0 1
x2
x +

x +
2
24
2 24
1

1
= 4 = .
1
2
2
1 (1

We have

2n

n=0

n! 2n(n1)/2

f (2) =

On the other hand,

n=0

n! 2n(n3)/2

=1+2+

2 1
1
.
+ +
n(n3)/2
2! 3! n=4 n! 2

1 1 1
1
=1+1+ + +
.
2! 3! n=4 n!
n=0 n!

e=

106

Since

n(n 3) > 0

n=4

n! 2n(n3)/2

d.

for

n 4,

1
n=4 n!

<

. Therefore,

we have

n! 2n(n3)/2
3
f (2) e < .
2

<

1
n!

for

n 4.

Hence

This time we have

(2)n
n(n1)/2
n=0 n! 2

(1)n
=
n(n3)/2
n=0 n! 2
2
(1)n
=12+ +
2! n=3 n! 2n(n3)/2

(1)n
=
n(n3)/2
n=3 n! 2
1
1
1
1
= + 2
5
+ 9
.
3! 2 4! 2 5! 2 6!

f (2) =

As

(n + 1)2n1 > 1 for n 3 , we have

too. Therefore,

14.

Estimate

f (2) <

x
0 e dx

Solution:

1
n! 2n(n3)/2

1
1
5
+ 2
= < 0.
3! 2 4!
32

with error less than

We have

ex = 1 + x +
for all

x.

x,

i.
ii.

iii.

un =

< 0 for n 3

0.01.

x2
xn
++
+
2!
n!

x4
x2n
+ + (1)n
+
2!
n!

and integration gives

Let

1
(n + 1)! 2(n+1)(n2)/2

Therefore,

ex = 1 x2 +
for all

ex dx = 2

22n+1
.
n! (2n + 1)

23 25
22n+1
+
+ + (1)n
+.
3 2! 5
n! (2n + 1)

Then:

22n+1
> 0 for all n 0 .
n! (2n + 1)
2n2 3n 1 > 0 for n 2 (n + 1)(2n + 1) > 22 (2n + 1)
un > un+1 for n 2 .
4n
2
lim un = lim (
) = 0.
n
n n! 2n + 1
un =

107

for

n 2.

Hence

The series satises the conditions of Alternating Series Test for

11!) = 32768/3586275 0.009 < 0.01,

n 2.

Since

it follows by the Alternating Series Estimate

that the sum

ex dx 2
2

approximates

15.

219
221
12223758182
23 25
+
+
+
=
0.89
3 2! 5
9! 19 10! 21 13749310575

x
0 e dx
2

with error less than

0.01.

Find the exact value of

Solution:

1
n
n=0 4 (2n + 1)

On one hand we have

1
(1/4)n (1/2)2n
(1/2)2n+1
=
=
=
2

n
n=0 2n + 1
n=0 2n + 1
n=0 2n + 1
n=0 4 (2n + 1)
3
5
(1/2)
(1/2)
= 2 (1/2 +
+
+ ) .
3
5

On the other hand we have

ln(1 + x) = (1)n+1
n=1
for

1 < x 1 .

In particular,

x = 1/2

xn
x2 x3 x4 x5
=x
+

n
2
3
4
5
gives

3
1
(1/2)2 (1/2)3 (1/2)4 (1/2)5
ln ( ) = ln (1 + ) = 1/2
+

+
,
2
2
2
3
4
5
and

x = 1/2

gives

1
1
(1/2)2 (1/2)3 (1/2)4 (1/2)5
ln ( ) = ln (1 ) = 1/2

.
2
2
2
3
4
5
Therefore

n=0

4n (2n + 1)

223 /(23

3
1
= ln ( ) ln ( ) = ln 3 .
2
2

108

Part 4: Vector Analysis

1.

Find the value of the line integral

(3x2 y 2 + y) dx + 2x3 y dy
C

where

is the cardioid

Solution:

r = 1 + cos

We use the Green's Theorem

where

parameterized counterclockwise.

M dx + N dy = (
R

N M

) dA
x
y

is the region enclosed by the simple closed curve

C.

Therefore

(3x2 y 2 + y) dx + 2x3 y dy = ( ( 2x3 y ) (3x2 y 2 + y)) dA


y
R x
C
= (6x2 y (6x2 y + 1)) dA
R

= dA
=
=

R
2

1+cos

0
2
0

r dr d

r2 r=1+cos
]
d
2 r=0

1 2
= (1 + 2 cos + cos2 ) d
2 0
1 2
1 + cos 2
=
(1 + 2 cos +
) d
2 0
2
1 3
= 2
2 2
3
= .
2

Remark We used the Circulation-Curl Form

of the Green's Theorem, but the computation is

exactly the same with the Flux-Divergence Form :

M dy N dx = (
R

109

M N
+
) dA
x
y

In fact, when expressed in terms of components and coordinates, both forms of the Green's
Theorem can be summarized and most easily remembered as

C
where

= A dx + B dy

= M dx + N dy ,

Remark following Example 30 in Part 2.

Indeed,

M
M
N
N
dx +
dy) dx + (
dx +
dy) dy
x
y
x
y

= M dy N dx,

then

d = dM dy dN dx = (
=

M
M
N
N
dx +
dy) dy (
dx +
dy) dx
x
y
x
y

N
M
N
M N
M
dx dy
dy dx =
dx dy +
dx dy = (
+
) dx dy
x
y
x
y
x
y

dx dx = 0 = dy dy
dx dy

now we must always get a

2.

The multiplication of

M
N
M
N
N M
dy dx +
dx dy =
dx dy +
dx dy = (

) dx dy ,
y
x
y
x
x
y

where we used

d = dA dx + dB dy .

then

d = dM dx + dN dy = (

and if

is a dierential form and

dierentials here are as described in the


if

= d

and

dy dx = dx dy .

Unlike in the case of change of variables,

under the double integral and we must not get rid of the sign.

F dr

Evaluate

where

F=
and

y
x
i+ 2
j
2
+ 9y
4x + 9y 2

4x2

is the unit circle parametrized in the counterclockwise direction.

Solution:

Observe that
curl

at all points
Consider

y
( 2
)
( 2
)
2
x 4x + 9y
y 4x + 9y 2
1
8x2
1
18y 2
= 2

=0
4x + 9y 2 (4x2 + 9y 2 )2 4x2 + 9y 2 4x2 + 9y 2

F=

(x, y) =/ (0, 0).

C0 r = (cos t)/2 i + (sin t)/3 j, 0 t 2 , the counterclockwise


4x2 + 9y 2 = 1. Let R be the region lying inside the

parametrization of the ellipse

unit circle and outside this ellipse.

Since curl

F=0

at all points of

Green's Theorem,

C0

Therefore by the generalized

y dx + x dy
2
2
C0 4x + 9y
t=2 (sin t)/3 d((cos t)/2) + (cos t)/2 d((sin t)/3)
=
t=0
cos2 t + sin2 t
1 2
1

=
dt = 2 = .
6 0
6
3

F dr =

F dr =
C

R, curl F dA = 0 .
R

110

3.

Find the surface area of the parameterized torus

r = (a + b cos u) cos v i + (a + b cos u) sin v j + b sin u k ,


0 u 2 , 0 v 2 ,

Solution:

where

0<b<a

are constants.

We have:

ru = b sin u cos v i b sin u sin v j + b cos u k


rv = (a + b cos u) sin v i + (a + b cos u) cos v j
ru rv = b(a + b cos u) cos v cos u i b(a + b cos u) sin v cos u j b(a + b cos u) sin u k
ru rv = b(a + b cos u)(cos2 v cos2 u + sin2 v cos2 u + sin2 u)1/2 = b(a + b cos u)
Therefore
Surface Area

= ru rv dA = b(a + b cos u) dA
=

where

R
2

R = {(u, v) 0 u 2

0
and

b(a + b cos u) du dv = ab 2 2 = 4 2 ab

0 v 2}.

111

4.
x2

Find the area of the portion


+ y 2 + z 2 = 4.

Solution:

of the cylinder

x2 + y 2 = 2y

that lies inside the sphere

F (x, y, z) = x2 + y 2 2y . Then the cylinder is given by F (x, y, z) = 0.


We will use the projection to the yz -plane. This projection is 2-1 from the portion of
the cylinder inside the sphere to the region R given by the inequalities 0 2y 4z 2
in the yz -plane, and by symmetry
Let

Surface Area of

where

p = i.

F
S = 2
dA
R F p

We have

F = 2xi + (2y 2)j


F = (4x2 + (2y 2)2 )1/2 = (4x2 + 4y 2 8y + 4)1/2 = 2
F p = F i = 2x

on

and therefore

Surface Area

F
2
1
dA = 2
dA = 2
dA
R F p
R 2x
R x

1
2y y 2 on S
= 2
dA
as x =
R
2y y 2

2
42y
2
1
4 2y

= 2
dz dy = 2 2
dy
2
0
42y
0
2y y
2y y 2
2 1

= 4 2 dy = 4 2 2 2 = 16 .
y
0
= 2

112

5.

Consider the parametrized surface

the area of the portion of the surface

S r = u2 i + 2 uvj + v 2 k, < u < , 0 v < .


S that lies inside the unit ball x2 + y 2 + z 2 1.

Find

x = u2 ,
y = 2uv and z = v 2 on the surface. Therefore,
x2 + y 2 + z 2 1 means (u2 )2 + ( 2uv)2 + (v 2 )2 1. In other words, (u2 + v 2 )2 1, or
u2 + v 2 1. Hence the part of the surface lying inside the sphere is the image of the
region R, the upper half of the unit disk, in the uv -plane.

Solution:

We have

To compute the surface area we rst compute:

ru = 2ui + 2vj

rv = 2uj + 2vk

ru ru = 2 2v 2 i 4uvj + 2 2k

ru ru = ((2 2v 2 )2 + (4uv)2 + (2 2)2 )1/2 = 2 2(u2 + v 2 )

Therefore,
Surface Area

= ru ru du dv
R

= 2 2 (u2 + v 2 ) du dv

= 2 2

2
0 r r dr d

1
= d
2 0

=
2
where we used the polar coordinates in the

Remark:

uv -plane.

There is a shorter way of solving this problem which does not use Calculus. The

given parametrization maps the upper half-uv -plane onto the half-cone given by the equation

y 2 = 2xz ,

and the conditions

x0

and

z0

u-axis).
positive x- and

in a one-to-one manner (except on the

This half-cone has its vertex at the origin, its axis lies along the bisector of the
z -axes, and it has an opening angel of 45 . The portion of this half-cone cut o by the unit
sphere is the lateral surface of a right cone with slant height
with area

r = / 2.
113

=1

and radius

r = 1/ 2,

hence

6.

Verify Stokes's Theorem for the vector eld F = y i + z k and the surface S , where S is the
z = x2 +y 2 satisfying z 3, with the unit normal vector eld n pointing

portion of the paraboloid


away from the

z -axis.

Solution:

is bounded by the curve

=
=

t=2
t=0
2
0

Cr=

3 cos t i

3 sin t j + 3 k, 0 t 2 .
Note that this parametrization is consistent with the direction of n. The circulation
of F around C is

F dr = (y i + z k) (dx i + dy j + dz k)
S

( 3 sin t i + 3 k) (d( 3 cos t) i + d( 3 sin t) j + d(3) k)

( 3 sin t)( 3 sin t) dt = 3

2
0

sin2 t dt = 3

f (x, y, z) = z x2 y 2 , and
f = 2x i 2y j + k points in the opposite direction to n, so we will choose the
minus sign from in the ux integral. The projection of S into the xy -plane is the
disk R = {(x, y) x2 + y 2 3}. Finally,

On the other hand, the paraboloid is a level surface of

F=

The ux of

= k .
z

across S is

f
S ( F) n d = R F k f dA
= (k) (2x i + 2y j k) dA
R

= dA = Area of R = ( 3)2 = 3 .
R

Hence

S ( F) n d =

F dr.
C

7.

Verify Divergence Theorem for the vector eld


{(x, y, z) x2 + y 2 + z 2 4} .

Solution:

F = xz i + yz j + z 3 k

and the region

F = (xz)/x + (yz)/y + (z 3 )/z = z + z + 3z 2 = 2z + 3z 2 ,

2
D F dV = D (2z + 3z ) dV

2
0

2
2
2
0 0 (2 cos + 3 cos ) sin d d d

96
cos2 ) sin d d
(8 cos +

5
0
0
64 2
128
=
d =
.

5 0
5

114

and

D =

S = {(x, y, z) x2 + y 2 + z 2 = 4}, we
divide it as the upper hemisphere S1 = {(x, y, z) x2 + y 2 + z 2 = 4 and x 0} and the
lower hemisphere S2 = {(x, y, z) x2 + y 2 + z 2 = 4 and x 0}, and project both onto
the disk R = {(x, y) x2 + y 2 4} in the xy -plane.

To compute the outward ux through the sphere

The sphere is a level surface of


in the same direction as

n,

f (x, y, z) = x2 + y 2 + z 2 . f = 2x i + 2y j + 2z k

the outward pointing unit normal vector eld on

we will choose the plus sign from

in the ux integral.

For the upper hemisphere we have

f
S F n d = R F k f dA
1

= (xz i + yz j + z 3 k)
R

2x i + 2y j + 2z k
dA
2z

= (x2 + y 2 + z 3 ) dA
R

= (x2 + y 2 + (4 x2 y 2 )3/2 ) dA
=
=

R
2

2
2 3/2
0 (r + (4 r ) ) r dr d

0
2
0

(4 +

32
64
) d = 8 +
.
5
5

A similar computation for the lower hemisphere gives

64
S F n d = 8 + 5 .
2
Therefore

S F n d = S F n d + S F n d
2

64
64
128
= (8 +
) + (8 +
)=
,
5
5
5
and

points

S F n d = D F dV

115

S,

so

Anda mungkin juga menyukai